feed twitter facebook

Best Blogger Tips

Chitika

4/15/09

sample question for Saudi Council for Health Specialties

Select the single best answer by blackening the circle of the correct answer [A,B,C,D, or E] on the answer sheet provided.

Q.1. A 30 years old man presented with generalized fatigue ability for 2 months. On examination he has found to have generalized lymph adenopathy. Differential diagnosis of this case includes all of the following except:
A. Lymphoma. T
B. HIV infection.
C. Hepatitis C virus infection.
D. Infectious mononucleosis.
E. Toxoplasmosis.
Q.2. Obstructed jaundice is associated with all of the following except:
A. Elevated indirect serum bilirubinaemia.
B. Elevated direct serum bilirubinaemia.
C. Presence of bilirubinuria.
D. Urobilinogenuria is almost always present.
E. Raised alkaline phosphatase.
Q.3. In rheumatoid arthritis all of the following are true except:
A. Causes destruction of articular cartilage.
B. Is frequently associated with the HLA antigen DR4.
C. Is equally common in males & females.
D. Is characterized by the presence of nodules.
E. Can involve any synovial joint in the body.
Q.4. Which of the following indicate pre-renal failure:
A. Cast in the urine.
B. Low urine osmolality [< 400 mosmmol/kg]
C. Low urine sodium concentration [< 20mmol/L]
D. Low free water excretion.
E. Microscopic hematuria.
Q.5. Hyponatremia occurs in the following conditions except:
A. Primary adrenocortical insufficiency.
B. Congestive heart failure.
C. Diabetic ketoacidosis with very high blood sugar.
D. Excessive production of vasopressin.
E. Diabetic insipidus.
Q.6. A 50 years old man was found unresponsive 3 hours after abdominal surgery. Arterial blood gas analysis on room air showed: pH – 7.2, PCO2 –65, HCO3 – 28, PO2 – 50, the most likely metabolic abnormality is:
A. Metabolic acidosis.
B. Respiratory & metabolic acidosis.
C. Metabolic acidosis.
D. Both respiratory acidosis & metabolic alkalosis.
E. Respiratory acidosis.
Q.7. A 35 years old lady, who acquired HIV infection after blood transfusion 10 years ago. Total CD4 lymphocyte count is < 100 cells/mm3. She is at high risk to develop any of the following problems except:
A. Tuberculosis.
B. CNS lymphoma.
C. Esophageal carcinoma.
D. Recurrent pneumonia.
E. Non-Hodgkin’s lymphoma.
Q.8. A 60 years old man presented with chest pain of 2 hours duration. ECG showed ST elevation in V1-V4 and frequent PVCs with a short run [6 beats] of ventricular tachycardia. Which of the following is an appropriate management?
A. Quinidine sulfates 200mg P.O. QID.
B. Digoxin 2mg IV push then 0.25mg IV OD for 3 days.
C. No anti arrhythmic therapy should be given until there is recurrent ventricular tachycardia.
D. Lidocane 75mg IV push followed continuous infusion at 2 mg/min.
E. Oxygen, Morphine sulfate and lasix.
Q.9. Which of the following is a usual feature of interstitial lung disease like fibrosing alveolitis:
A. Fever.
B. Hemolysis.
C. Generalized wheezes.
D. Purulent sputum.
E. Fine crepitations.
Q.10. Which of the following statements regarding the management of an acute sever asthma attack is correct:
A. Measurement of arterial blood gasses is important only in those who fail to respond to treatment.
B. Inhale glucocorticoids are indicated in large doses.
C. Rise of PaCO2 from low to normal value is a sign of good response.
D. IV aminophyline has a narrow therapeutic range.
E. Fluid restriction is indicated to relieve pulmonary edema.
Q.11. A young man took refrigerated food. 3 hours later he developed watery diarrhea. Which of the following organisms most likely caused the diarrhea?
A. Shigella.
B. Salmonella.
C. Campylobacter.
D. Staphylococci.
E. Giardia.
Q.12. In mitral stenosis, the following may be seen except?
A. P. mitral on ECG.
B. Double contour of right border of the heart [due to left atrial enlargement].
C. Left ventricular failure.
D. Right ventricular failure.
E. Atrial fibrillation.
Q.13. Giardiasis?
A. Involves mainly the terminal ileum & colon.
B. It is more common in hypogammaglobinimic patient.
C. Liver abscess is a recognized complication.
D. Is best treated by mebendazole.
E. May cause lower GI bleeding.
Q.14. The commonest cause of acute pancriatitis in adult in our community is?
A. Unknown.
B. Parasites.
C. Mumps virus.
D. Billiary stones.
E. Ethanol.
Q.15. All of the following are true about grave’s disease except?
A. Opthalmopathy invariably responds to anti-thyroid therapy.
B. Pre-tibial myxedema is a feature.
C. Atrial fibrillation may occur.
D. Patient may have difficulty in climbing stairs.
E. There is often a bruit heard over the thyroid gland.
Q.16. A 65 years old man present with difficulty in swallowing food. Causes of this condition include all of the following except?
A. Carcinoma of esophagus.
B. Reflux oesophagitis.
C. Achalasia of the cardia.
D. Motor neuron disease.
E. Herpes oesophagitis.
Q.17. Plasma ferritin is a reflection of?
A. Red cell mass.
B. Total body iron stores.
C. Bone marrow iron stores.
D. Hepatic iron content.
E. None of the above.
Q.18. A 26 years old man presented with headache & fatigue. Investigation revealed Hb 8gm/dl, MCV-85, Reticulocytes 10%. The following investigations are useful except?
A. Coomb’s test.
B. Sickling test.
C. Serum bilirubin.
D. Serum iron.
E. Hemoglobin electrophoresis.
Q.19. The following may induce seizures except:
A. Hypoxia.
B. Hypourecaemia.
C. Hypocalcaemia.
D. Hyponatremia.
E. Hypocalcaemia.
Q.20. A20 years old man presented with one-day history of headache & fever. On examination, he was restless, avoid light & his neck was resistant to flexion. Which of the following lines of management is correct?
A. X-ray of cervical spine.
B. Electroencephalogram.
C. Antinuclear antibodies.
D. Phenytoin.
E. None of the above.
Q.21. The following disease are recognized cause of high output cardiac Failure except?
A. Paget’s disease.
B. Thyrotoxicosis.
C. Hypertension.
D. Arterio-venous fistula.
E. Iron deficiency anemia.
Q.22. All of the pharmacological agents may be useful in the treatment of the acute manifestations of anaphylaxis except?
A. Diphenhydramine.
B. Hydralazine.
C. Oxygen.
D. Epinephrine.
E. Aminophyline.
Q.23. In ulcerative colitis?
A. Fistula is common finding.
B. Involve small& large bowel.
C. Pyoderma gangrenosum can occur.
D. Toxic mega colon should be diagnosed by colonoscopy.
E. Usual manifestation is diarrhea alternating with constipation.
Q.24. Appropriate management of patients with adrenal insufficiency secondary to hypo-pituitarism include all of the following except?
A. Glucocorticoids replacement.
B. Mineral corticoid therapy.
C. Steroid coverage for surgical procedures.
D. Instruction in self-injection of soluble glucocorticoids.
E. Wearing of identification bracelet bearing medical information.


Q.25. When a disease lasts 2 to 3 weeks with a fatality rate of 80%, then its incidence?
A. Is higher than its prevalence.
B. Is equal to its prevalence.
C. Is lower than its prevalence.
D. Has no relationship to its prevalence.
E. Is one half of its prevalence?
Q.26. The most common complication & the most common cause of death in transplant recipients is:
A. Rejection.
B. Steroid overdose.
C. Cardiac failure.
D. Toxic effects of immunosuppressive drugs.
E. Infection.
Q.27. A 40-years old women presented with bloody nipple discharge, no Family history of the same problem. The commonest cause is?
A. Breast carcinoma.
B. Intraductal papilloma.
C. Glactocoele.
D. Cystic hyperplasia.
E. Paget’s disease of the breast.
Q.28. Management of burn wound includes the following except?
A. Early debridement.
B. Skin grafting.
C. Escharotomy.
D. Tetanus immunization.
E. Topical application of penicillin powder.
Q.29. Clinical feature of toxic goiter includes the following except?
A. Restlessness.
B. Arrhythmias.
C. Hoarseness of voice.
D. Clammy palms.
E. Tremors of fingers.
Q.30. A 20 year's old male patient had a stab wound in the neck. All are true except?
A. A lacerated carotid artery [ies] is [are], better ligated because of sever bleeding.
B. False aneurysm may develop.
C. Injured major lymphatics are ligated.
D. Tracheal injury is closed primarily & tracheostomy performed.
E. Subcutaneous emphysema may performed.
Q.31. Veins of the legs affected in varicose veins are the following Except?
A. Long saphenous veins.
B. Short saphenous veins.
C. Inter communicating veins.
D. Venae perforantes.
E. Popliteal vein.
Q.32. Complication of surgical excision of abdominal aneurysm include the following except?
A. Paralytic ileus.
B. Ischemia of lower extremities.
C. Paraplegia.
D. Hepatic failure.
E. Renal failure.
Q.33. Torsion of testis. All are correct except?
A. Is treated with large doses of antibiotics.
B. Is torsion of the spermatic cord?
C. Is the result of “ Bell clapper” deformity?
D. Occurs spontaneously in young males.
E. May occur as a result of physical & sexual activities.
Q.34. Presentation of benign prostatic hyperplasia include the following except?
A. Nocturia.
B. Diminished size & force of urinary stream.
C. Prostatitis.
D. Hematuria.
E. Urinary retention.
Q.35. The pain of uretric calculus is referred?
A. To the flank.
B. To the tip of the penis.
C. To the anal area.
D. To the region of the bladder.
E. To the sacral area.
Q.36. In tracheo-esophageal fistula, management include the following Except?
A. Insertion of a chest tube.
B. Performing a gastrostomy.
C. Pulmonary toileting.
D. Insertion of a naso-esophageal stump catheter.
E. Intravenous antibiotics.
Q.37. On a ventilator, recommended tidal volume is?
A. 500cc.
B. 1 liter.
C. 1.5 liters.
D. 2 liters.
E. 2.5 liters
Q.38. A 70 years old women presented with a 3 days history of Perforated duodenal ulcer. She was febrile, semi comatose & dehydrated on admission. The best treatment is?
A. Transfuse with blood, rehydrated & perform vagotomy & drainage urgently.
B. Insert a nasogastric tube & connect to suction, hydrate the patient, give systemic antibiotic & observe.
C. Insert a nasogastric tube & connect to suction, hydrate the patient, give systemic antibiotic & and perform plication of the perforation.
D. Hydrate the patient, give blood, give systemic antibiotics& perform hemigastrectomy.
E. None of the above.
Q.39. A 2 months old child was found to be jaundice, to have an enlarged cirrhotic liver & ascites. The most likely diagnosis is?
A. Gilbert’s disease.
B. Criglar-Najjar disease.
C. Dubin-Johnson syndrome.
D. Congenital Billiary atresia.
E. Sickle cell disease.
Q.40. An important clinical finding in acute pancriatitis is?
A. Rebound tenderness.
B. Presence of some relief on bending forward or on maintaining a sitting position.
C. Guarding of the abdomen.
D. Hypoactive bowel sound.
E. Abdominal pain.
Q.41.Two weeks after cholecystectomy a patient present with progressive jaundice & possible ascites. The most indicate investigation is?
A. CT scan.
B. Intravenous cholangiography.
C. Blood for hepatitis.
D. Needle liver biopsy.
E. Endoscopic retrograde cholangio-pancreatography [ERCP]
Q.42. In blunt trauma to the abdomen the most commonly injured organ Is?
A. The small intestine.
B. The spleen.
C. The bladder.
D. The kidney.
E. The liver.
Q.43. A 20 years old boy sustained a blunt trauma to the abdomen in an automobile accident. Intraperitoneal bleeding is suspected. The most informative investigation is?
A. Plain X-ray of the abdomen.
B. Diagnostic peritoneal lavage.
C. PCV estimation.
D. Intravenous urogram.
E. Celiac angiography.
Q.44. The following are complication of laparoscopic cholecystectomy Except?
A. Bile leak.
B. Persistent pneumoperitoneum.
C. Shoulder tips pain.
D. Ascites.
E. Supraumblical incisional hernia.
Q.45. Differential diagnosis of acute appendicitis includes the Following except?
A. Mittleschmerz disease.
B. Carcinoma of caecum.
C. Pelvic inflammatory disease.
D. Diverticulitis.
E. Mesenteric lymphadenitis.
Q.46. Complication of colostomy include the following except?
A. Malabsorption of water.
B. Retraction.
C. Prolapse.
D. Excoriation of skin.
E. Obstruction.
Q.47. The greatest metabolic activity of thyroid hormone is due to?
A. Thyroxin.
B. Monoiodotyrosine.
C. Diiodotryrosine.
D. Triiodothyronine.
E. Thyroglobulin.
Q.48. Complication of osteomyelitis include the following except?
A. Septic arthritis.
B. Arrest of bone growth.
C. Pyomyositis.
D. Septicemia.
E. Destruction of epiphyseal plate.
Q.49. An incomplete lesion on the side of the spinal cord produces?
A. Decorticate rigidity.
B. Decerebrate rigidity.
C. Absence of spino-ciliary cortex.
D. Brown-sequard syndrome.
A. E. All the above.
Q.50. The standard deviated is:
A. A measure of location.
B. Equivalent to the range.
C. A measure of normality of the data.
D. A measure of variability.
E. A measure of central tendency.

Q.51. Factors that is important in the patho-physiology of pelvic inflammatory disease include the following except?
A. Menstruation.
B. Sperm.
C. Cervical mucus.
D. Uterine fibroids.
E. Anaerobic bacteria.
Q.52. All of the following factors is necessary for the menstruation to occur except?
A. Hypothalamic releasing hormones.
B. An endometrium response to sex steroids.
C. Gonadotrophin.
D. Patent fallopian tube.
E. Ovarian steroid hormones.
Q.53. Gross microscopic feature of myomas include all of the following Except?
A. A whorled like appearance.
B. Bundles of smooth muscle fibers.
C. High mitosis figure.
D. Central degeneration.
E. Peripheral vascularity.

Q.54. S/S associated with a hydatidiform mole include all of the following except?
A. First trimester bleeding.
B. A uterus large than the expected gestational age.
C. Hypothyroidism.
D. Pre-eclampsia at 14 weeks gestation.
E. Nausea & vomiting.
Q.55. Correct statements regarding the respiratory distress syndrome Include all of the following except?
A. There is deficiency of surfactant in the lung.
B. Hypoxia & acidosis stimulate the production of the lung phospholipids.
C. It is the most common problem of the pre-term neonate.
D. There is a ground-glass appearance on X-ray.
E. Positive & expiratory pressure is useful treatment modalities.
Q.56. A women States that her last menstruation period was 7 weeks ago & that she had several days of light bleeding & lower abdominal discomfort. She ahs previously had a positive home pregnancy test. Which of the following studies would you request at this time?
A. Human chorionic Gonadotrophin.
B. Human placental lactogen.
C. Progesterone.
D. Estriol.
E. Prolactin.
Q.57. The main health hazard of menopause is?
A. Cardiovascular disease.
B. Pelvic relaxation.
C. Endometrial cancer.
D. Depression.
E. Osteoporosis.
Q.58. Postpartum hemorrhage could be a reasonable possibility in all of the following situations except?
A. Hydromnios.
B. Triplets.
C. Prolong labour.
D. Erythroblastosis.
E. Thrombocytopenia.
Q.59. Which of the following S/S needs not to be reported immediately as a potential danger signal in a pregnant woman?
A. Vaginal bleeding.
B. Sever headache.
C. Swelling of the ankle & feet.
D. Blurring of the vision.
E. Escape of fluid from the vagina.
Q.60. Deficiency of which of the following substances is likely in a pregnant vegetarian?
A. Folic acid.
B. Vitamin B12.
C. Protein.
D. Calcium.
E. Iron.
Q.61. Naegele’s for estimating a women’s due date is based on all of the following factors except?
A. Regular monthly menstrual cycles.
B. A pregnancy of 280 days.
C. Ovulation about day 14.
D. Cycle regulation with birth control pills prior to conception.
E. Conception at mid-cycle.
Q.62. Appropriate screening tests in an early, uncomplicated pregnancy include all of the following except?
A. Repeat serum HCG level.
B. Hemoglobin.
C. Serology.
D. Cervical cytology.
E. Blood type & Rh.

Q.63. Factors that can contribute to an acute urinary Tract infection during pregnancy, delivery, or the puerperium include all
of the following except?
A. Compression of the ureter by the large uterus at the pelvic brim.
B. Increased urethral tone & peristalsis.
C. Asymptomatic bacteriuria.
D. Decreased bladder sensitivity after epidural anesthesia.
E. Bladder catheterization following delivery.
Q.64. Appropriate form of management for the pre-eclemptic patient [B.P of 140/95 & 1+ proteinuria] include all of the following except?
A. Bed rest.
B. Non-stress tests.
C. Hospitalization.
D. Serial sonography of the fetus.
E. Diuretics.
Q.65. The most accurate & definitive documentation of fetal life is made by?
A. Fetoscope.
B. Leopoid maneuver.
C. Pelvic X-ray.
D. Real time.
E. Doppler ultrasound.
Q.66. The average length of the menstrual cycle is?
A. 22 days.
B. 25 days.
C. 28 days.
D. 35 days.
E. 38 days.
Q.67. All of the following factors is associated with an increased risk of pelvic inflammatory disease except?
A. Onset of intercourse at age 15.
B. An elective abortion.
C. Oral contraceptive.
D. Hystero-salphingography.
E. Used of copper IUD.
Q.68. All of the following procedures can aid in the diagnosis of pelvic inflammatory disease except?
A. Laparoscopy.
B. Ultrasound.
C. Coldocentesis.
D. Rectal examination.
E. Hystero-salphingography.
Q.69. If a women present with chronic yeast infection, it is important to elicit a history of all of the following except?
A. Diabetes.
B. Pregnancy.
C. Use of antibiotics.
D. Use of oral contraceptive.
E. Use of vinegar douches.
Q.70. Atrophic vaginitis would be expected in all of the following Clinical situation except?
A. Menopause.
B. Lactation.
C. Oral contraceptive use.
D. Surgical castration in young women.
E. Psuedo-menopause during endometriosis therapy.
Q.71. For women over 35 years of age with abdominal uterine bleeding, the most accurate diagnostic procedure is?
A. Basal body temperature.
B. Endometrial biopsy.
C. Fractional dilatation & curettage.
D. Hysteroscopy.
E. Hormone therapy.
Q.72. All of the therapeutic procedures are recommended for ectopic pregnancies except?
A. Salpingectomy.
B. Salphingo-oophorectomy.
C. Linear Salpingectomy.
D. Segmental resection of the portion of the tube containing the ectopic pregnancy.
E. Milking the enrapture pregnancy from the tube.
Q.73. What is the most accurate method of diagnosing an ectopic pregnancy?
A. Pelvic ultrasound.
B. Culdocentesis.
C. Laparoscopy.
D. Measurement of serial B-HCG levels.
E. Endometrial biopsy.
Q.74. All the following factors are evidence that ovulation has Occurred except?
A. Rise in basal body temperature.
B. Pregnancy.
C. Progesterone level above 3 ng/ml.
D. Secretary endometrium.
E. The occurrence of menses.
Q75. Incidence is calculated by the number of?
A. Old case during the study period.
B. New case during the study.
C. New case at a point in time.
D. Old case at a point in time.
E. Existing cases at a study period.
Q.76. Which of the following would be the most valuable measurement in the monitoring the clinical response of a patient with acute hemorrhage?
A. Hematocrit.
B. Hemoglobin.
C. Vital signs.
D. Platelet count.
E. Volume of blood lost.
Q.77. The most important cause of epistaxis in children is?
A. Trauma.
B. Polyp.
C. Sinusitis.
D. Thrombocytopenia.
E. Dry air.
Q.78. The combination of Ataxia, a positive Babinski sing & absence of deep tendon reflexes of ankle indicate?
A. Ataxia telangiectasia.
B. Friedreich’ ataxia.
C. Ganglioneuroblastoma.
D. Parinaud syndrome [pinealoma]
E. Lead poisoning.
Q.79. All of the following are true in pyloric stenosis except?
A. Incidence in male is higher than in female.
B. Onset is generally late in the first month of life.
C. Jaundice occurs in association.
D. Vomitus is bile stained.
E. Appetite is good.
Q.80. Children are expected to walk alone without support by age?
A. 6 months.
B. 9 months.
C. 15 months.
D. 18 months.
E. 20 months.
Q.81. Gastric lavage is contraindicated after Ingestion of?
A. Aspirin.
B. Diazepam.
C. Castor beans.
D. Drain-cleaning solution.
E. Vitamin D.
Q.82. Apgar score takes all of the following except?
A. Blood pressure.
B. Heart rate.
C. Respiratory effort.
D. Color.
E. Muscles tone.
Q.83. Which statement is true about bacterial pneumonia in children?
A. It occurs following most upper respiratory infections.
B. It is unusual event in normal children.
C. Staphylococcus pneumonia is the leading cause.
D. Physical examination in infant is diagnostic.
E. The mortality rate of pneumococcal pneumonia approaches 3-5%.
Q.84.The treatment of children with cleft palate may be complicated by?
A. Speech disorder.
B. Dental caries.
C. Malocclusion.
D. Otitis media.
E. Obstructed apnea.
Q.85. A new born Exhibits respiratory distress, low blood pressure, hyper resonance & diminished breath sounds over side of the chest. The most likely diagnosis is?
A. Staphylococcus pneumonia.
B. Hyaline membrane disease.
C. Pneumothorax.
D. Primary atelectasis.
E. Diaphragmatic hernia.
Q.86. Central cyanosis is present in the following except?
A. Transposition of great arteries.
B. Tricuspid atresia.
C. Total anomalous pulmonary venous drainage.
D. Isolated pulmonary stenosis.
E. Pulmonary atresia.
Q.87. Each of the following statements regarding oral rehydration therapy is true except?
A. It can be used to treat acute diarrhea in patients of all age.
B. Oral rehydration solution should contain 5 to 7% glucose.
C. Oral rehydration solution containing 45 mEq/l of sodium & those with 75 mEq/l of sodium generally is equally efficacious.
D. It has been used successfully to treat patients with Hyponatremia.
E. All of the above.


Q.88. A healthy infant is born at 30 weeks gestation. Size & development is appropriate for gestational age. Her mother wants to breast feed her. A true statement about breast-feeding for the preterm infant is?
A. The volume of the breast milk will be greater in mothers who have breast feed previously.
B. Human milk production is optimal if the mother expresses milk once daily until the infant begins nursing.
C. After only 30 weeks gestation, maternal milk production rarely is sufficient to permit breast-feeding.
D. The mother should begin expressing milk as soon as possible.
E. None of the above.
Q.89. Examination of the CSF of a 2 years old child supports the diagnosis of bacterial meningitis. The child is moderately dehydrated. The best approach to prescribing fluid therapy for this child would be to?
A. Restrict fluid to prevent the syndrome of inappropriate antidiuretic hormone [SIADH].
B. Treat the dehydration immediately.
C. Consider the possibility of SIADH only if the child’s urine sodium concentration is greater than 60 mEq/l.
D. Treat the dehydration after antibiotic therapy has been provided for 24 hours.
E. None of the above.
Q.90. A 2 years old patient is brought to you because of snoring & Disturb sleep. You suspect upper airway obstruction with sleep apnea. Each of the following statements about this condition is true except?
A. Death can occur.
B. Cardiac failure may occur.
C. Systemic hypertension is an associated finding.
D. Pulmonary hypertension is an irreversible side effect.
E. None of the above.
Q.91. Of the following problems, the most common bleeding diathesis that would present as recurring epistaxis is?
A. Hemophilia A [factor VIII]
B. Hemophilia B [factor IX]
C. Hemophilia C [factor XI]
D. Hageman factor deficiency.
E. Von Willebrand disease.
Q.92. The control of blood sugar in juvenile diabetes mellitus is usually achieved by?
A. Short & intermediate insulin.
B. Long acting insulin.
C. Intermediate insulin.
D. Oral hypoglycemic.
E. Short acting insulin.
Q.93. The best source of iron for the 3 months old infant is?
A. Iron fortified cereals.
B. Yellow vegetable.
C. Fruits.
D. Breast milk.
E. 2% low fat cow’s milk.
Q.94. Among the following, the least likely risk factor for hearing loss is:
A. A maternal history of use of Phenytoin during pregnancy.
B. A family history of hearing impairment.
C. Craniofacial abnormality.
D. Birth weights less than 1500 Gms.
E. Neonatal hyperbilirubinemia.
Q.95. Protein losing gastro-enteropathy has been associated with all of the following except?
A. Cow milk protein.
B. Lymphangiectasia.
C. Granulomatous disease of intestine.
D. Ulcerative colitis.
E. Soy protein formula.
Q.96. Cellulites occurring about the face in young children [6 to 24 months] & associated with fever & a purple skin discoloration is most often caused by?
A. Group A beta hemolytic streptococci.
B. Haemophilus influenzae.
C. Streptococcus pneumoniae.
D. Staphylococcus aureus.
E. None of the above.
Q.97. All of the following might be possible etiologies for discoloration of the teeth of a 15 months old infant except?
A. Iron medication.
B. Dental decay.
C. Tetracycline given to mother during pregnancy.
D. Lactose free formula.
E. Important fluoride administration.
Select the single best answer by blackening the circle of the correct answer [A,B,C,D, or E] on the answer sheet provided.
Q.1. A 30 years old man presented with generalized fatigue ability for 2 months. On examination he has found to have generalized lymph adenopathy. Differential diagnosis of this case includes all of the following except:
A. Lymphoma. T
B. HIV infection.
C. Hepatitis C virus infection.
D. Infectious mononucleosis.
E. Toxoplasmosis.
Q.2. Obstructed jaundice is associated with all of the following except:
A. Elevated indirect serum bilirubinaemia.
B. Elevated direct serum bilirubinaemia.
C. Presence of bilirubinuria.
D. Urobilinogenuria is almost always present.
E. Raised alkaline phosphatase.

Q.3. In rheumatoid arthritis all of the following are true except:
A. Causes destruction of articular cartilage.
B. Is frequently associated with the HLA antigen DR4.
C. Is equally common in males & females.
D. Is characterized by the presence of nodules.
E. Can involve any synovial joint in the body.
Q.4. Which of the following indicate pre-renal failure:
A. Cast in the urine.
B. Low urine osmolality [< 400 mosmmol/kg]
C. Low urine sodium concentration [< 20mmol/L]
D. Low free water excretion.
E. Microscopic hematuria.
Q.5. Hyponatremia occurs in the following conditions except:
A. Primary adrenocortical insufficiency.
B. Congestive heart failure.
C. Diabetic ketoacidosis with very high blood sugar.
D. Excessive production of vasopressin.
E. Diabetic insipidus.
Q.6. A 50 years old man was found unresponsive 3 hours after abdominal surgery. Arterial blood gas analysis on room air showed: pH – 7.2, PCO2 –65, HCO3 – 28, PO2 – 50, the most likely metabolic abnormality is:
A. Metabolic acidosis.
B. Respiratory & metabolic acidosis.
C. Metabolic acidosis.
D. Both respiratory acidosis & metabolic alkalosis.
E. Respiratory acidosis.
Q.7. A 35 years old lady, who acquired HIV infection after blood transfusion 10 years ago. Total CD4 lymphocyte count is < 100 cells/mm3. She is at high risk to develop any of the following problems except:
A. Tuberculosis.
B. CNS lymphoma.
C. Esophageal carcinoma.
D. Recurrent pneumonia.
E. Non-Hodgkin’s lymphoma.
Q.8. A 60 years old man presented with chest pain of 2 hours duration. ECG showed ST elevation in V1-V4 and frequent PVCs with a short run [6 beats] of ventricular tachycardia. Which of the following is an appropriate management?
A. Quinidine sulfates 200mg P.O. QID.
B. Digoxin 2mg IV push then 0.25mg IV OD for 3 days.
C. No anti arrhythmic therapy should be given until there is recurrent ventricular tachycardia.
D. Lidocane 75mg IV push followed continuous infusion at 2 mg/min.
E. Oxygen, Morphine sulfate and lasix.
Q.9. Which of the following is a usual feature of interstitial lung disease like fibrosing alveolitis:
A. Fever.
B. Hemolysis.
C. Generalized wheezes.
D. Purulent sputum.
E. Fine crepitations.
Q.10. Which of the following statements regarding the management of an acute sever asthma attack is correct:
A. Measurement of arterial blood gasses is important only in those who fail to respond to treatment.
B. Inhale glucocorticoids are indicated in large doses.
C. Rise of PaCO2 from low to normal value is a sign of good response.
D. IV aminophyline has a narrow therapeutic range.
E. Fluid restriction is indicated to relieve pulmonary edema.
Q.11. A young man took refrigerated food. 3 hours later he developed watery diarrhea. Which of the following organisms most likely caused the diarrhea?
A. Shigella.
B. Salmonella.
C. Campylobacter.
D. Staphylococci.
E. Giardia.
Q.12. In mitral stenosis, the following may be seen except?
A. P. mitral on ECG.
B. Double contour of right border of the heart [due to left atrial enlargement].
C. Left ventricular failure.
D. Right ventricular failure.
E. Atrial fibrillation.
Q.13. Giardiasis?
A. Involves mainly the terminal ileum & colon.
B. It is more common in hypogammaglobinimic patient.
C. Liver abscess is a recognized complication.
D. Is best treated by mebendazole.
E. May cause lower GI bleeding.
Q.14. The commonest cause of acute pancriatitis in adult in our community is?
A. Unknown.
B. Parasites.
C. Mumps virus.
D. Billiary stones.
E. Ethanol.


Q.15. All of the following are true about grave’s disease except?
A. Opthalmopathy invariably responds to anti-thyroid therapy.
B. Pre-tibial myxedema is a feature.
C. Atrial fibrillation may occur.
D. Patient may have difficulty in climbing stairs.
E. There is often a bruit heard over the thyroid gland.
Q.16. A 65 years old man present with difficulty in swallowing food.
Causes of this condition include all of the following except?
A. Carcinoma of esophagus.
B. Reflux oesophagitis.
C. Achalasia of the cardia.
D. Motor neuron disease.
E. Herpes oesophagitis.
Q.17. Plasma ferritin is a reflection of?
A. Red cell mass.
B. Total body iron stores.
C. Bone marrow iron stores.
D. Hepatic iron content.
E. None of the above.
Q.18. A 26 years old man presented with headache & fatigue. Investigation revealed Hb 8gm/dl, MCV-85, Reticulocytes 10%. The following investigations are useful except?
A. Coomb’s test.
B. Sickling test.
C. Serum bilirubin.
D. Serum iron.
E. Hemoglobin electrophoresis.
Q.19. The following may induce seizures except:
A. Hypoxia.
B. Hypourecaemia.
C. Hypocalcaemia.
D. Hyponatremia.
E. Hypocalcaemia.
Q.20. A20 years old man presented with one-day history of headache & fever. On examination, he was restless, avoid light & his neck was resistant to flexion. Which of the following lines of management is correct?
A. X-ray of cervical spine.
B. Electroencephalogram.
C. Antinuclear antibodies.
D. Phenytoin.
E. None of the above.
Q.21. The following disease are recognized cause of high output cardiac Failure except?
A. Paget’s disease.
B. Thyrotoxicosis.
C. Hypertension.
D. Arterio-venous fistula.
E. Iron deficiency anemia.
Q.22. All of the pharmacological agents may be useful in the treatment of the acute manifestations of anaphylaxis except?
A. Diphenhydramine.
B. Hydralazine.
C. Oxygen.
D. Epinephrine.
E. Aminophyline.
Q.23. In ulcerative colitis?
A. Fistula is common finding.
B. Involve small& large bowel.
C. Pyoderma gangrenosum can occur.
D. Toxic mega colon should be diagnosed by colonoscopy.
E. Usual manifestation is diarrhea alternating with constipation.
Q.24. Appropriate management of patients with adrenal insufficiency secondary to hypo-pituitarism include all of the following except?
A. Glucocorticoids replacement.
B. Mineral corticoid therapy.
C. Steroid coverage for surgical procedures.
D. Instruction in self-injection of soluble glucocorticoids.
E. Wearing of identification bracelet bearing medical information.
Q.25. When a disease lasts 2 to 3 weeks with a fatality rate of 80%, then its incidence?
A. Is higher than its prevalence.
B. Is equal to its prevalence.
C. Is lower than its prevalence.
D. Has no relationship to its prevalence.
E. Is one half of its prevalence?
Q.26. The most common complication & the most common cause of death in transplant recipients is:
A. Rejection.
B. Steroid overdose.
C. Cardiac failure.
D. Toxic effects of immunosuppressive drugs.
E. Infection.

Q.27. A 40-years old women presented with bloody nipple discharge, no Family history of the same problem. The commonest cause is?
A. Breast carcinoma.
B. Intraductal papilloma.
C. Glactocoele.
D. Cystic hyperplasia.
E. Paget’s disease of the breast.
Q.28. Management of burn wound includes the following except?
A. Early debridement.
B. Skin grafting.
C. Escharotomy.
D. Tetanus immunization.
E. Topical application of penicillin powder.
Q.29. Clinical feature of toxic goiter includes the following except?
A. Restlessness.
B. Arrhythmias.
C. Hoarseness of voice.
D. Clammy palms.
E. Tremors of fingers.
Q.30. A 20 year's old male patient had a stab wound in the neck. All are true except?
A. A lacerated carotid artery [ies] is [are], better ligated because of sever bleeding.
B. False aneurysm may develop.
C. Injured major lymphatics are ligated.
D. Tracheal injury is closed primarily & tracheostomy performed.
E. Subcutaneous emphysema may performed.
Q.31. Veins of the legs affected in varicose veins are the following Except?
A. Long saphenous veins.
B. Short saphenous veins.
C. Inter communicating veins.
D. Venae perforantes.
E. Popliteal vein.
Q.32. Complication of surgical excision of abdominal aneurysm include the following except?
A. Paralytic ileus.
B. Ischemia of lower extremities.
C. Paraplegia.
D. Hepatic failure.
E. Renal failure.
Q.33. Torsion of testis. All are correct except?
A. Is treated with large doses of antibiotics.
B. Is torsion of the spermatic cord?
C. Is the result of “ Bell clapper” deformity?
D. Occurs spontaneously in young males.
E. May occur as a result of physical & sexual activities.
Q.34. Presentation of benign prostatic hyperplasia include the following except?
A. Nocturia.
B. Diminished size & force of urinary stream.
C. Prostatitis.
D. Hematuria.
E. Urinary retention.
Q.35. The pain of uretric calculus is referred?
A. To the flank.
B. To the tip of the penis.
C. To the anal area.
D. To the region of the bladder.
E. To the sacral area.
Q.36. In tracheo-esophageal fistula, management include the following Except?
A. Insertion of a chest tube.
B. Performing a gastrostomy.
C. Pulmonary toileting.
D. Insertion of a naso-esophageal stump catheter.
E. Intravenous antibiotics.
Q.37. On a ventilator, recommended tidal volume is?
A. 500cc.
B. 1 liter.
C. 1.5 liters.
D. 2 liters.
E. 2.5 liters
Q.38. A 70 years old women presented with a 3 days history of Perforated duodenal ulcer. She was febrile, semi comatose & dehydrated on admission. The best treatment is?
A. Transfuse with blood, rehydrated & perform vagotomy & drainage urgently.
A. Insert a nasogastric tube & connect to suction, hydrate the patient, give systemic antibiotic & observe.
B. Insert a nasogastric tube & connect to suction, hydrate the
C. patient, give systemic antibiotic & and perform plication of the perforation.
D. Hydrate the patient, give blood, give systemic antibiotics& perform hemigastrectomy.
E. None of the above.

Q.39. A 2 months old child was found to be jaundice, to have an enlarged cirrhotic liver & ascites. The most likely diagnosis is?
A. Gilbert’s disease.
B. Criglar-Najjar disease.
C. Dubin-Johnson syndrome.
D. Congenital Billiary atresia.
E. Sickle cell disease.
Q.40. An important clinical finding in acute pancriatitis is?
A. Rebound tenderness.
B. Presence of some relief on bending forward or on maintaining a sitting position.
C. Guarding of the abdomen.
D. Hypoactive bowel sound.
E. Abdominal pain.

Q.41. Two weeks after cholecystectomy a patient present with progressive jaundice & possible ascites. The most indicate
investigation is?
A. CT scan.
B. Intravenous cholangiography.
C. Blood for hepatitis.
D. Needle liver biopsy.
E. Endoscopic retrograde cholangio-pancreatography [ERCP]
Q.42. In blunt trauma to the abdomen the most commonly injured organIs?
A. The small intestine.
B. The spleen.
C. The bladder.
D. The kidney.
E. The liver.
Q.43. A 20 years old boy sustained a blunt trauma to the abdomen in an automobile accident. Intraperitoneal bleeding is suspected. The most informative investigation is?
A. Plain X-ray of the abdomen.
B. Diagnostic peritoneal lavage.
C. PCV estimation.
D. Intravenous urogram.
E. Celiac angiography.
Q.44. The following are complication of laparoscopic cholecystectomy Except?
A. Bile leak.
B. Persistent pneumoperitoneum.
C. Shoulder tips pain.
D. Ascites.
E. Supraumblical incisional hernia.
Q.45. Differential diagnosis of acute appendicitis includes the Following except?
A. Mittleschmerz disease.
B. Carcinoma of caecum.
C. Pelvic inflammatory disease.
D. Diverticulitis.
E. Mesenteric lymphadenitis.
Q.46. Complication of colostomy include the following except?
A. Malabsorption of water.
B. Retraction.
C. Prolapse.
D. Excoriation of skin.
E. Obstruction.
Q.47. The greatest metabolic activity of thyroid hormone is due to?
A. Thyroxin.
B. Monoiodotyrosine.
C. Diiodotryrosine.
D. Triiodothyronine.
E. Thyroglobulin.
Q.48. Complication of osteomyelitis include the following except?
A. Septic arthritis.
B. Arrest of bone growth.
C. Pyomyositis.
D. Septicemia.
E. Destruction of epiphyseal plate.
Q.49. An incomplete lesion on the side of the spinal cord produces?
A. Decorticate rigidity.
B. Decerebrate rigidity.
C. Absence of spino-ciliary cortex.
D. Brown-sequard syndrome.
A. E. All the above.
Q.50. The standard deviated is:
A. A measure of location.
B. Equivalent to the range.
C. A measure of normality of the data.
D. A measure of variability.
E. A measure of central tendency.
Q.51. Factors that is important in the patho-physiology of pelvic inflammatory disease include the following except?
A. Menstruation.
B. Sperm.
C. Cervical mucus.
D. Uterine fibroids.
E. Anaerobic bacteria.
Q.52. All of the following factors is necessary for the menstruation to occur except?
A. Hypothalamic releasing hormones.
B. An endometrium response to sex steroids.
C. Gonadotrophin.
D. Patent fallopian tube.
E. Ovarian steroid hormones.
Q.53. Gross microscopic feature of myomas include all of the following Except?
A. A whorled like appearance.
B. Bundles of smooth muscle fibers.
C. High mitosis figure.
D. Central degeneration.
E. Peripheral vascularity.
Q.54. S/S associated with a hydatidiform mole include all of the following except?
A. First trimester bleeding.
B. A uterus large than the expected gestational age.
C. Hypothyroidism.
D. Pre-eclampsia at 14 weeks gestation.
E. Nausea & vomiting.
Q.55. Correct statements regarding the respiratory distress syndrome Include all of the following except?
A. There is deficiency of surfactant in the lung.
B. Hypoxia & acidosis stimulate the production of the lung phospholipids.
C. It is the most common problem of the pre-term neonate.
D. There is a ground-glass appearance on X-ray.
E. Positive & expiratory pressure is useful treatment modalities.
Q.56. A women States that her last menstruation period was 7 weeks ago & that she had several days of light bleeding & lower abdominal discomfort. She ahs previously had a positive home pregnancy test. Which of the following studies would you request at this time?
A. Human chorionic Gonadotrophin.
B. Human placental lactogen.
C. Progesterone.
D. Estriol.
E. Prolactin.
Q.57. The main health hazard of menopause is?
A. Cardiovascular disease.
B. Pelvic relaxation.
C. Endometrial cancer.
D. Depression.
E. Osteoporosis.
Q.58. Postpartum hemorrhage could be a reasonable possibility in all of the following situations except?
A. Hydromnios.
B. Triplets.
C. Prolong labour.
D. Erythroblastosis.
E. Thrombocytopenia.
Q.59. Which of the following S/S needs not to be reported immediately as a potential danger signal in a pregnant woman?
A. Vaginal bleeding.
B. Sever headache.
C. Swelling of the ankle & feet.
D. Blurring of the vision.
E. Escape of fluid from the vagina.
Q.60. Deficiency of which of the following substances is likely in a pregnant vegetarian?
A. Folic acid.
B. Vitamin B12.
C. Protein.
D. Calcium.
E. Iron.
Q.61. Naegele’s for estimating a women’s due date is based on all of the following factors except?
A. Regular monthly menstrual cycles.
B. A pregnancy of 280 days.
C. Ovulation about day 14.
D. Cycle regulation with birth control pills prior to conception.
E. Conception at mid-cycle.
Q.62. Appropriate screening tests in an early, uncomplicated pregnancy include all of the following except?
A. Repeat serum HCG level.
B. Hemoglobin.
C. Serology.
D. Cervical cytology.
E. Blood type & Rh.
Q.63. Factors that can contribute to an acute urinary Tract infection during pregnancy, delivery, or the puerperium include all of the following except?
A. Compression of the ureter by the large uterus at the pelvic brim.
B. Increased urethral tone & peristalsis.
C. Asymptomatic bacteriuria.
D. Decreased bladder sensitivity after epidural anesthesia.
E. Bladder catheterization following delivery.
Q.64. Appropriate form of management for the pre-eclemptic patient [B.Pof 140/95 & 1+ proteinuria] include all of the following except?
A. Bed rest.
B. Non-stress tests.
C. Hospitalization.
D. Serial sonography of the fetus.
E. Diuretics.
Q.65. The most accurate & definitive documentation of fetal life is made by?
A. Fetoscope.
B. Leopoid maneuver.
C. Pelvic X-ray.
D. Real time.
E. Doppler
ultrasound.
Q.66. The average length of the menstrual cycle is?
A. 22 days.
B. 25 days.
C. 28 days.
D. 35 days.
E. 38 days.
Q.67. All of the following factors is associated with an increased risk of pelvic inflammatory disease except?
A. Onset of intercourse at age 15.
B. An elective abortion.
C. Oral contraceptive.
D. Hystero-salphingography.
E. Used of copper IUD.
Q.68. All of the following procedures can aid in the diagnosis of pelvic inflammatory disease except?
A. Laparoscopy.
B. Ultrasound.
C. Coldocentesis.
D. Rectal examination.
E. Hystero-salphingography.
Q.69. If a women present with chronic yeast infection, it is important to elicit a history of all of the following except?
A. Diabetes.
B. Pregnancy.
C. Use of antibiotics.
D. Use of oral contraceptive.
E. Use of vinegar douches.
Q.70. Atrophic vaginitis would be expected in all of the following Clinical situation except?
A. Menopause.
B. Lactation.
C. Oral contraceptive use.
D. Surgical castration in young women.
E. Psuedo-menopause during endometriosis therapy.
Q.71. For women over 35 years of age with abdominal uterine bleeding, the most accurate diagnostic procedure is?
A. Basal body temperature.
B. Endometrial biopsy.
C. Fractional dilatation & curettage.
D. Hysteroscopy.
E. Hormone therapy.
Q.72. All of the therapeutic procedures are recommended for ectopic pregnancies except?
A. Salpingectomy.
B. Salphingo-oophorectomy.
C. Linear Salpingectomy.
D. Segmental resection of the portion of the tube containing the ectopic pregnancy.
E. Milking the enrapture pregnancy from the tube.
Q.73. What is the most accurate method of diagnosing an ectopic pregnancy?
A. Pelvic ultrasound.
B. Culdocentesis.
C. Laparoscopy.
D. Measurement of serial B-HCG levels.
E. Endometrial biopsy.
Q.74. All the following factors are evidence that ovulation has Occurred except?
A. Rise in basal body temperature.
B. Pregnancy.
C. Progesterone level above 3 ng/ml.
D. Secretary endometrium.
E. The occurrence of menses.

Q75. Incidence is calculated by the number of?
A. Old case during the study period.
B. New case during the study.
C. New case at a point in time.
D. Old case at a point in time.
E. Existing cases at a study period.
Q.76. Which of the following would be the most valuable measurement in the monitoring the clinical response of a patient with acute hemorrhage?
A. Hematocrit.
B. Hemoglobin.
C. Vital signs.
D. Platelet count.
E. Volume of blood lost.
Q.77. The most important cause of epistaxis in children is?
A. Trauma.
B. Polyp.
C. Sinusitis.
D. Thrombocytopenia.
E. Dry air.
Q.78. The combination of Ataxia, a positive Babinski sing & absence of deep tendon reflexes of ankle indicate?
A. Ataxia telangiectasia.
B. Friedreich’ ataxia.
C. Ganglioneuroblastoma.
D. Parinaud syndrome [pinealoma]
E. Lead poisoning.
Q.79. All of the following are true in pyloric stenosis except?
A. Incidence in male is higher than in female.
B. Onset is generally late in the first month of life.
C. Jaundice occurs in association.
D. Vomitus is bile stained.
E. Appetite is good.
Q.80. Children are expected to walk alone without support by age?
A. 6 months.
B. 9 months.
C. 15 months.
D. 18 months.
E. 20 months.
Q.81. Gastric lavage is contraindicated after Ingestion of?
A. Aspirin.
B. Diazepam.
C. Castor beans.
D. Drain-cleaning solution.
E. Vitamin D.
Q.82. Apgar score takes all of the following except?
A. Blood pressure.
B. Heart rate.
C. Respiratory effort.
D. Color.
E. Muscles tone.
Q.83. Which statement is true about bacterial pneumonia in children?
A. It occurs following most upper respiratory infections.
B. It is unusual event in normal children.
C. Staphylococcus pneumonia is the leading cause.
D. Physical examination in infant is diagnostic.
E. The mortality
rate of pneumococcal pneumonia approaches 3-5%.
Q.84.The treatment of children with cleft palate may be complicated by?
A. Speech disorder.
B. Dental caries.
C. Malocclusion.
D. Otitis media.
E. Obstructed apnea.

Q.85. A new born Exhibits respiratory distress, low blood pressure, hyper resonance & diminished breath sounds over side of the chest. The most likely diagnosis is?
A. Staphylococcus pneumonia.
B. Hyaline membrane disease.
C. Pneumothorax.
D. Primary atelectasis.
E. Diaphragmatic hernia.
Q.86. Central cyanosis is present in the following except?
A. Transposition of great arteries.
B. Tricuspid atresia.
C. Total anomalous pulmonary venous drainage.
D. Isolated pulmonary stenosis.
E. Pulmonary atresia.

Q.87. Each of the following statements regarding oral rehydration therapy is true except?
A. It can be used to treat acute diarrhea in patients of all age.
B. Oral rehydration solution should contain 5 to 7% glucose.
C. Oral rehydration solution containing 45 mEq/l of sodium & those with 75 mEq/l of sodium generally is equally efficacious.
D. It has been used successfully to treat patients with Hyponatremia.
E. All of the above.
Q.88. A healthy infant is born at 30 weeks gestation. Size & development is appropriate for gestational age. Her mother wants to breast feed her. A true statement about breast-feeding for the preterm infant is?
A. The volume of the breast milk will be greater in mothers who have breast feed previously.
B. Human milk production is optimal if the mother expresses milk once daily until the infant begins nursing.
C. After only 30 weeks gestation, maternal milk production rarely is sufficient to permit breast-feeding.
D. The mother should begin expressing milk as soon as possible.
E. None of the above.
Q.89. Examination of the CSF of a 2 years old child supports the diagnosis of bacterial meningitis. The child is moderately dehydrated. The best approach to prescribing fluid therapy for this child would be to?
A. Restrict fluid to prevent the syndrome of inappropriate antidiuretic hormone [SIADH].
B. Treat the dehydration immediately.
C. Consider the possibility of SIADH only if the child’s urine sodium concentration is greater than 60 mEq/l.
D. Treat the dehydration after antibiotic therapy has been provided for 24 hours.
E. None of the above.
Q.90. A 2 years old patient is brought to you because of snoring & Disturb sleep. You suspect upper airway obstruction with sleep apnea. Each of the following statements about this condition is true except?
A. Death can occur.
B. Cardiac failure may occur.
C. Systemic hypertension is an associated finding.
D. Pulmonary hypertension is an irreversible side effect.
E. None of the above.
Q.91. Of the following problems, the most common bleeding diathesis that would present as recurring epistaxis is?
A. Hemophilia A [factor VIII]
B. Hemophilia B [factor IX]
C. Hemophilia C [factor XI]
D. Hageman factor deficiency.
E. Von Willebrand disease.
Q.92. The control of blood sugar in juvenile diabetes mellitus is usually achieved by?
A. Short & intermediate insulin.
B. Long acting insulin.
C. Intermediate insulin.
D. Oral hypoglycemic.
E. Short acting insulin.
Q.93. The best source of iron for the 3 months old infant is?
A. Iron fortified cereals.
B. Yellow vegetable.
C. Fruits.
D. Breast milk.
E. 2% low fat cow’s milk.
Q.94. Among the following, the least likely risk factor for hearing loss is:
A. A maternal history of use of Phenytoin during pregnancy.
B. A family history of hearing impairment.
C. Craniofacial abnormality.
D. Birth weights less than 1500 Gms.
E. Neonatal hyperbilirubinemia.
Q.95. Protein losing gastro-enteropathy has been associated with all of the following except?
A. Cow milk protein.
B. Lymphangiectasia.
C. Granulomatous disease of intestine.
D. Ulcerative colitis.
E. Soy protein formula.
Q.96. Cellulites occurring about the face in young children [6 to 24 months] & associated with fever & a purple skin discoloration is most often caused by?
A. Group A beta hemolytic streptococci.
B. Haemophilus influenzae.
C. Streptococcus pneumoniae.
D. Staphylococcus aureus.
E. None of the above.
Q.97. All of the following might be possible etiologies for discoloration of the teeth of a 15 months old infant except?
A. Iron medication.
B. Dental decay.
C. Tetracycline given to mother during pregnancy.
D. Lactose free formula.
E. Important fluoride administration.
MCQ’S FOR MEDICAL PRACTETIONER
Q-1- 25 yrs old female pt C/O sudden attack of sever headaches a days ago with vomiting,diplopia & disturb sleep for fear of Brain tumor Ct scan, MRI, CSF are free, what is the most likely diagnosis?
1. Hypochondrial disorder.
2. Somatization disorder
3. Anxiety
4. -
5. -

Q-2- With regards to cephalosporin allergy, all of the following are true, except?
1. Skin test for allergy is available.
2. Drug sensitivity can occur
3. Cefacolor frequently use can be associated with serum sickness like syndrome
4. Cefaroxime can cause choledo-cholelithiasis like picture
5. -
Q-3- Regarding Phenytoin side effects, all of the following are true, except?
1. Can cause gingival hypertrophy
2. Macrocytosis
3. Ataxia
4. Osteomalasia
5. Osteoporosis
Q-4- Regarding Giardia infestation, all of the following are true, except?
1. Cause liver disease
2. Protein losing enteropathy
3. Proliferation in the terminal ileum.
4. -
5. -
Q-5- Female pt develop solitary benign tumor, H/P shows fissure in its nucleus, the tumor is most probable?
1. Benign
2. -
3. -
4. -
5. -
Q-6- About measles, all of the following are true, except?
1. Highly contagious
2. It slut with upper respiratory infection
3. Active immunization at the age of 9 months
4. Epidemic range of age is 3-5 yrs.
5. Symptomatic traetment is required
Q-7- With regarding to Thyrotoxic Goiter, all of the following are true, except?
1. Bruit can be osculated on thyroid
2. With treatment improves optico-------
3. Climbing stair may be difficult
4. Peritibial myxedema may occur
5. Atrial fibrillation may occur

Q-8- Hyponatremia can occur with, all of the following, except?
1. Increased vasopressin
2. CCF
3. Prolong diuretic use
4. Diabetes insipidus
5. -
Q-9- all of the following can cause seizure, except?
1. Hyponatremia
2. Hypourecaemia
3. Hypokalaemia
4. Hypoxia
5. -
Q-10- With regards of oral contraceptive, all of the following are true, except?
1. Thrombo-embolic tendency rather than atherosclerotic lesion are the cause of embolic manifestation.
2. It decreased incidence of ovarian tumor.
3. Increased incidence of Ca breast in pills user than non pills user.
4. Pt with previous pills user have more incidence to develop DVT
5. -
Q-11 – The pt develops rapidly progressive pelvic mass without bleeding the most probable diagnosis is?
1. Leiomyosarcoma
2. Fibroma
3. Adenocarcinoma.
4. -
5. -
Q-12 – All of the following are true, except?
1. Asymptomatic bacteriuria can affect about 10% of female.
2. Chemotherapy of bacteriuria can prophylaxis against pregnancy complication.
3. Female are more susceptible than males to UTI.
4. Up to 40% pt develop acute pyelonephritis without treatment
5. -
Q-13 – With regards to indication of CT scan in dementia, which one is not true?
1. Pt < 60 yrs of age
2. After head injury
3. H/O dementia 3 yrs ago
4. -
5. -
Q-14 – Serum ferritin reflect?
1. Total body iron
2. B/M iron store
3. Serum iron
4. -
5. -
Q-15 – In pre-renal failure?
1. Low urinary obstruction
2. Low cons of urinary Na
3. Free water excretion
4. -
5. -
Q-16 – Hashimotos thyroiditis, all of the following are true, except?
1. Decreased T4
2. Increased TSH
3. Increased cholesterol
4. Thyroid antibodies
5. -
Q-17 – Female patient presented with acute diarrhea after intake of refrigerated food, the following organism may be the causative agent?
1. Shigella
2. Salmonella
3. Campylobacter
4. Vibro cholera
5. E coli

Q-18 – With regards to Herpes Simplex virus type-I?
1. It is the commonest viral infection of the oral cavity.
2. Can give rise to gingio-stomatitis
3. In primary infection there is always systemic involvement
4. May be present with tonsillitis without oral lesion.-
5. -
Q-19 – In scabies infection, all of the following are true, except?
1. Rarely involve head & neck
2. 5% Lindane is effective
3. benzyl-benzo-benzoate is equally effective as lindane
4. itching occur 4-7 days after infestation
5. -
Q-20 – Interstitial lung disease is characterized by?
1. Hemoptysis
2. Fine basal rales
3. -
4. -
5. -
Q-21 – A 65yrs old patient with difficulty in swallowing food, all of the following are true, except?
1. Acid ingestion
2. Ca Esophagus
3. Reflex oesophagitis
4. Achalasia of esophagus
5. Motor neuron disease
Q-22 – Male patient with recent extensive MI present 3 days later with shock, pan systolic murmur, the most probable diagnosis is?
1. Ruptured VSD
2. Ruptured of papillary nodules
3. Ventricular aneurysm
4. Papillary muscles dysfunction
5. -
Q-23 – Female patient with incompetent Cervix Os seek medical acre with pregnancy 8 wks, the most probable management is?
1. Cervical circulage at the time of silting
2. Bed rest with circulage at 16-18 wks
3. Hormones therapy
4. -
5. -
Q-24 – WPW syndrome ECG finding, all of the following are true, except?
1. Delta waves
2. Long P-R interval
3. Wide QRS complex
4. -
5. -
Q-25 – Generalized lymphadenopathy are present in all of the following except?
1. Hepatitis C virus
2. Aids
3. CMV
4. Lymphoma
5. -
Q-26 – The most common cause of precousious puberty in girls is?
1. Idiopathic
2. Ovarian tumor
3. Increased B-HCG
4. Over estrogen production
5. -
Q-27 – Female patient presented 8-hrs after surgery, cyanosis, dysnoea, oligurea, ABG shows,
O2-82, CO2-23, HCO3-23, pH-72, the pt has?
1. Metabolic acidosis
2. Respiratory acidosis
3. Mixed respiratory alkalosis & metabolic acidosis
4. Mixed respiratory acidosis & metabolic alkalosis
5. Non of the above
Q-28 – With regards sub-lingual nitro glyceride, all of the following are true, except?
1. It causes pooling of venous blood
2. Increased coronary blood flow
3. Peak pharmacological action at 4-8min
4. Cause met-hemoglobinemia
5. Duration of action 10-30 min
Q-29 – With regards to H2 receptors antagonist in treatment of duodenal ulcer, all of the following are true, Except?
1. Alternate dose have a same effect as daily dose
2. No role of it in prophylaxis from NASID induced duodenal ulcer
3. Long term prophylactic therapy is not recommended
4. -
5. -
Q-30 – Infective Endocarditis is characterized by all of the following, except?
1. Osler’s node in finger pulp.
2. Anemia
3. Immune complex phenomenon [small emboli, ms hematuria]
4. Splinter hemorrhages on nail bed.
5. -
Q-31 – Rheumatoid Arthritis have, all of the following character, except?
1. Equal incidence in male & female
2. Cause joint deformity
3. Have positive R.A factor
4. -
5. -
Q-32 – Patient with acute MI with frequent PVCS the first line of management is?
1. Lignocain bolus [Lignocain 50mg bolus I/V repeat twice at 5 min interval if necessary
2. Than slow Lignocain infusion]
3. -
4. -
5. -
Q-33 – In patient with Aids depressed total T-cell count, predisposed to all of the following except?
1. Esophageal Carcinoma.
2. Monolial chest infection
3. TB
4. Lymphoma
5. -
Q-34 – With regards to stroke, all of the following are true, except?
1. In male pt 75 yrs old with AF should be contained on prophylactic anti-coagulant
2. AF with valvular heart disease can cause stroke
3. AF without valvular heart disease in 75 yrs old pt can lead to stroke
4. -
5. -
Q-35 – Acute sever asthma all of the following are true, Except?
1. ABC is not necessary done except if there is no response to primary treatment.
2. Aminophyline has a narrow safe therapeutic effect level
3. Inhalation corticosteroid should be given a large dose.
4. -
5. -
Q-36 – With regards to Eclampsia, all of the following are true, except?
1. Its common in primigravida
2. The use of barrier method of contraception can decreased the incidence.
3. It's more frequent in multigravida with same partner than those who have multiple partners.
4. -
5. -


Q-37 – Vestibular neuritis cause all except?
1. Dizziness
2. Vertigo
3. Mild to moderate neural deafness[acustic neuritis]
4. -
5. -
Q-38 – 30 yrs old male patient c/o dizziness especially when he moves his head from side to side,
which is the most appropriate cause?
1. acute lybrynthitis
2. vestibular neuritis
3. Miner's disease [vertigo, tinnitus, deafness]
4. Orthostatic hypotension
5. -
Q-39 – Young child with unilateral nasal discharge, foul odor, since one week, the diagnosis is?
1. Foreign body
2. Viral infection
3. Bacterial infection
4. Tumor
5. Polyp unilateral
Q-40 – Young child with grayish white membrane over the tonsils, fever, diagnosis may be all except?
1. Acute follicular tonsillitis
2. Vincent angina
3. Infectious mononucleosis
4. Candidiasis
5. Erysipelas
Q-41 – Use of vasoconstrictor nasal drop for long time may cause?
1. Allergic rhinitis
2. Perforated septum
3. Rebound phenomenon
4. Rhinitis Sciacca
5. -
Q-42 – Child with sever sensory neural nerve deafness you will do?
1. Place him in deaf school
2. Learn him lip reading
3. Use hearing aids
4. Learn him alone up to 6 yrs
5. -
Q-43 – Middle age male come with pain in right eye, saying he can not open his eye due to purulent discharge, diagnosis is?
1. Bacterial conjunctivitis
2. Viral conjunctivitis
3. Glaucoma
4. Uveitis
5. Pelberitis
Q-44 – 40 year's old male come with acute excruciating pain in one eye, nausea, vomiting, headache, the diagnosis is?
1. Acute glaucoma
2. Iriti sepis cleritis
3. Corneal ulcer
4. -
5. -
Q-45 – retinal changes in eye you will refer the pt to ophthalmologist?
1. Waxy changes at periphery
2. Plots & dots of Hemorrhages
3. Moculopathy
4. Exudates on retina
5. Nipping of retinal vessels


Q-46 – 35 yrs female come with sever pain in eye circum corneal injection with blurred vision, the probable diagnosis is?
1. Iritis
2. Episcleritis
3. Corneal ulcer
4. Glaucoma
5. Foreign body
Q-47 – Action of abducent nerve right & left, action of trochlear nerve right & left?
1. Diplopia
2. Convergence strabismus
3. -
4. -
5. -
Q-48 - Patient with ulcerative colitis, recurrent backache, when examined by ophthalmologist there was?
1. Photophobia on ophthalmoscope
2. Dilated pupil
3. Retinal haze
4. -
Q-49 – Metronidazole is the treatment of choice in all except?
1. E. Histolytica
2. Trichomonas vaginalis
3. Shigella vaginitis
4. Pseudo-membrane ulcerative colitis
5. -
Q-50 – Suicidal tendency is more in?
1. 30 yrs old female with previous tried for suicide
2. 50 yrs old male with previous tried for suicide
3. 35 yrs old male recently diagnosed as Aids
4. 25 yrs old female effected with RA
5. -
Q-51 - Middle age women with c/o anxiety about her & children health & worried about her job, on investigation her children are completely normal & she does well in her job, she has?
1. Generalized anxiety disease
2. Depression
3. Hypochondriasis
4. Schizophrenia
5. Somatic s/s of anxiety
Q-52 – Schizophrenia will be diagnosed by?
1. Primary delusion
2. Gustatory hallucination
3. Auditory hallucination
4. Somatization disorder[ common complains of female, pain, vomiting, nausea, headache,
5. Menstrual irregularities with -ive investigation]
Q-53 – 30 yrs old male anxious, worry, has insomnia, trouble his wife & impotence, there was good marital history before, on examination he is normal, what is the probable diagnosis?
1. Depression
2. Anxiety
3. Inhibit desire of sex
4. Conversion
5. -
Q-54 – Female C/O having cancer stomach & visited many doctors & did many investigations, barium, U/S all are normal, diagnosis is?
1. Anxiety
2. Hypochondria sis
3. Somatization
4. Schizophrenia
5.
Q-55 – Female patient C/O paralysis of both left limbs & paresthesia when examined she was stretched over bed & neurologist found every thing is normal & was told that she always beated by husband, diagnosis is?
1. Psychogenic paralysis
2. Hypochondria sis
3. Hysterical
4. Anxiety
5. Obsessive compulsive syndrome




Q-56 – Patient has a lesion on front of fore arm and was diagnosed lichen planus you will search for lesion also in?
1. mouth
2. extensor surface of knee & elbow
3. buttock
4. face
5. chest
Q-57 - Koebner’s phenomenon is found in all, except?
1. Lichen planus
2. Warts
3. Psoriasis
4. Moluscum contagiosum
5. Acne vulgaris
Q-58 – Acne rosicia is treated by?
1. Amoxicillin
2. Cephalosporin
3. Penicillin
4. clindamycin
5. Low dose tetracycline & erythromycin
Q-59 – H. Pylori treated with?
1. Omeprazole + amoxicillin + clarithromycin
2. Omeprazole + amoxicillin
3. Omeprazole + amoxicillin + flagyl
4. Amoxil + flagyl
5. Septran + Amoxil + flagyl
Q-60 – About Erythema multiformi true is?
1. female > male
2. affect face only
3. caused by virus
4. localized disease
5. buccal mucosa never involved
Q-61 – All causes Itching except?
1. Lichen planus
2. Eczema
3. Drug allergy
4. Herpetic form Erythema
5. -
Q-62 – Zry $ multiple ulcer seen in, except?
1. Zry $
2. Chanceroid
3. Bechet’s disease
4. Granuloma inguinalae
5. Lymphogranuloma venerum
Q-63 – Female with dysmenorrhea treated with mefanemic acid, no response 6 months, you will do?
1. stop mefenamic acid & start diclofen Na
2. continue mefenamic acid for further 6 months
3. give mefenamic acid + hyocine compound
4. put on contraceptive pills
5. injectable analgesic
Q-64 – Contraceptive pills is useful true except?
1. No thromboembolic complication
2. Can be use as treatment in dysfunctional uterine bleeding
3. Can be used in old age
4. -
Q-65 – Pregnant women 30 wks with brucellosis you will do?
1. Give symptomatic treatment till delivery
2. Give doxycyclin
3. give tetracycline
4. –-
Q-66 – Pregnant women 11 wks c/o nausea & vomiting once a day she is on iron therapy, best to do?
1. Stop iron
2. Give maxolon
3. Admitted to hospital
4. No treatment
5. -


Q-67 – When couple come to you searching for fertility?
1. You will ask them about sexual technique
2. Do laparoscopy to find out any abnormality & may be treated
3. Varicocele dose not cause infertility
4. -
5. -
Q-68 – Amenorrhea for 6m the most serous diagnosis can be done by?
1. Abdominal ultrasound
2. Serum B-HCG
3. Plane x-ray
4. CBC
5. Urine D/R
Q-69 – Married female searching for fertility for 18 yrs, she has amenorrhea 6 weeks with low Abdominal pain is examination left abdominal mass, best to do?
1. Serum B-HCG
2. Culdocentesis
3. Abdominal ultrasound
4. Plane X-ray pelvis
5. CT scan
Q-70 – New born with jaundice 2 day, wt 38 kg, normal baby with Hb-15gm%, retic - 4%, indirect Coomb's test negative, there is difference b/w baby & mother blood group diagnosis is?
1. Rh incompatibility
2. ABO incompatibility
3. Thalassemia
4. Septicemia
5. Sickle cell anemia
Q-71 – Normal healthy baby with jaundice at 3rd day, TB- 11mgdl, diagnosis is?
1. Hepatitis
2. Obstructive jaundice
3. Criglar najar syndrome
4. Dubin jonson syndrome
5. -
Q-72 – Baby with, drowsy, convulsion, depressed fontanels, no neck stiffness, you will do?
1. CBC
2. CSF
3. Blood C/S
4. X-ray chest
5. Urine D/R
Q-73 – Acid base balance, normal value?
1. PO2 90-105
2. PCO2 35-45
3. PH 734-742
4. HCO3 38-42
5. -

Q-74 – A young baby c/o redness, edema or pain in arm with fever, all of the following are true, except?
1. Staphlococcus is a likely the organism
2. It may leads to deep abscess
3. Lymphagenitis & lymphadenopathy may occur at hot side
4. Local antibiotic is sufficient for treatment
5. -
Q-75 – A young child referred by his mother c/o tonic colonic convulsion, the best anti Convulsion to give?
1. Phenytoin
2. barbiturate
3. diazepam
4. oral muscle relaxant
5. analgesic

Q-76 – To differentiate b/w depression & dementia is?
1. Withdrawal from society
2. Decreased attention
3. Headache
4. Loss of wt
5. -


Q-77 – Young child referred c/o wetting his cloth for days, he is controlled 4 yrs ago, what you will do?
1. Urine analysis & c/s
2. Cystoscopy
3. Ultrasound
4. Cystogram
5. Urine D/R
Q-78 – When lactic acid increased in the body try to decreased acidosis
1. by excretion of CO2 by lungs
2. by excretion of lactic acid by kidney
3. metabolism of lactic acid by liver
4. metabolism of chloride by kidney
5. -
Q-79 – Acne vulgaris moderated or sever is treated by?
1. Petiniac acid local
2. Isoretinoin oral
3. benzyl peroxide cream
4. -
5. -
Q-80 – Pitting of finger nail seen in all except?
1. Psoriasis
2. Marked underlying malignancy
3. Lichen planus
4. Trauma
5. Alopecia areata
Q-81 – Baby with fever 38.4 C, distress, retraction sub costal & intercostals after URTI 2 days, cough with sound on examination breathing sound resembles passing of air in URT?
1. Viral pneumonia
2. Broncholitis
3. Bacterial pneumonia
4. Asthma
5. Foreign body

Q-82 – Anti-coagulant in lactating female?
1. Heparin
2. warfarin
3. Asprin
4. -
5.
Q-83 – Female complain of vulvitis , itching, crude thick discharge, diagnosis is?
1. Candidiasis
2. Chlamydia
3. Allergic
4. Trichomonas
5. Bacterial
Q-84 – Female with thin homogenous vaginal discharge without itching, diagnosis is?
1. Candidiasis
2. Chlamydia
3. Viral
4. Trichomonas
5. Bacterial
Q-85 – 60 yrs old male with chronic bronchial asthma on cortisone inhalation therapy, you want to add theophylin your fear of?
1. Gastric upset
2. Cardiac arrhythmia
3. -
4. -
5. -
Q-86 – Young male child complain of piteacheal ecchymosis, platelet decreased and hemoglobin decreased, treated by?
1. Prednisolone
2. Blood transfusion
3. Platelet transfusion
4. -
5. -
Other Questions=
Q1 – Patient present with left thigh inflexion, adduction & internal rotation, what is the diagnosis?
A. Impacted acetabular fracture
B. Anterior hip dislocation
C. Posterior hip dislocation
D. Sub capital fracture of the femur
E. Sacro-iliac separation
Q-2 – Hypokalaemia may be seen in all of the following except? Metabolic acidosis [In metabolic alkalosis not in m/b acidosis]
A. Furosemide therapy
B. Renal tubular acidosis
C. Hyper Aldosteronism
D. vomiting & diarrhea
Q-3 – 30 years old man came with complain of upper abdominal pain & dyspepsia, which of the following does not support the diagnosis of peptic ulcer?
A. Hunger pain
B. Heart burn
C. Epigastric mass
D. Epigastric pain relationship with meals[ Dyspepsia]
E. History of hematemesis
Q-4 – 82 years old male patient came with complain of acute urine retention, the best management for this patient is?
A. To take him to OT as an emergency & do the prostectomy.
B. Admission, investigation which include Cystoscopy, then possible TUR.
C. Give antibiotic b/c this retention may be due to infection & ask him to come back the clinic.
D. Empty the bladder by Foleys catheter & tell him to come back.
E. Insert Foleys catheter, send him home, ask him to come back the clinic later.
Q-5 – Stress ulceration is seen in all of the following except?
A. Burns
B. CNS lesion
C. Alcohol ingestion
D. Salicylate ingestion
E. Penicillin administration
Q-6 – Which of the following statement is true in pregnancy with DM?
A. There is increased risk of oligo Hydromnios
B. Insulin requirement remain unchanged in pregnancy
C. In the new born hypoglycemia may occur after delivery
D. Rarely cause fetal death
E. It can cause hyper calcemia in new born
Q-7 – A 10 years old girl present with 2 days history of fever & 4 cm mass tender & fluctuant left anterior cervical lymph node, the most likely diagnosis is?
A. Hodgkin’s disease
B. Acute lymphoblastic leukemia
C. Histocytosis
D. Acute bacterial lymphadenitis
E. Metastatic neuroblastoma
Q-8 – Which of the following statement best characterized the diagnosis of UTI in children?
A. The presence of fever localized infection to renal parenchyma
B. Diagnosis is likely if there is pyurea more then 10 WBC / hpf.
C. Diagnosis is likely if sterile specimen show 100,000 organism of a single species
D. Diagnosis is difficult because the typical causative organism grow poorly in culture
E. Vpsko uretric reflex localized infection to lower UTI.
Q-9 – Which of the following S/S should not relate to teething is?
A. Diarrhea
B. Rhinorrhea
C. Decrease appetite
D. Irritability
E. Fever of 39 C
Q-10 – Fecal leucocytes are not found in increased number in patient with?
A. Giardiasis
B. Shigellosis
C. Idiopathic ulcerative colitis
D. Clindamycin associated colitis
E. Enterocolitis
Q-11 – Known consequences of pelvic inflammatory diseases include all of the following except?
A. Dyspareunia
B. Ectopic pregnancy
C. Endometriosis
D. Infertility
E. Menstrual disturbance
Q-12 – Healthy 5 years old girl, sudden onset of hematuria which persists for 2 days, all of the following are reasonable immediate steps in the evaluation of the patient except?
A. Hemoglobin electrophoresis to exclude hemoglobinopathies
B. Cystoscopy to establish the site of bleeding
C. BUN & Cr measurement
D. Urine culture
E. Reveal U/S to access renal anatomy.
Q-13 – In infant 6m with sepsis, most likely diagnosis is?
A. Listeria cytogenous
B. Hemophilus influenzea
C. B Streptococcus hemolyticus
D. Staphlococcus aureus
E. E coli
Q-14 – UTI in children diagnosed by?
A. Isolation of organism with colony count 100,000 of the same organism.
B. -
C. -
D. -
E. -
Q-15 – Hypokalaemia occurs in all of the following except?
A. Metabolic acidosis
B. Acute tubular necrosis
C. Diuretic therapy.
D. Hyper aldosteronism
E. Cushing syndrome
Q-16 – In ARF all of the following are present except?
A. Uremia
B. Phosphataemia
C. Increased potassium in serum
D. Decreased calcium in serum
E. Decreased bicarbonate in serum
Q-17 – Patient with partial thickness of burn?
A. It is sensitive
B. It is insensitive
C. It will change to slough within 2-3 wks
D. Need split graft
E. Need free flap
Q-18 – Female 6 years child from Jezan with hematuria all investigation needed except?
A. Hb sickling test
B. Hb electrophoresis
C. Urine examination
D. U/S abdomen to see changes in glomerulous
E. Cystoscopy
Q-19 – A male boy came to you with C/O cola color urine, with streptococcal throat infection 3 weeks ago, what is best for diagnosed a post streptococcal glomerulonephritis?
A. RBC cast
B. Increased creatinine
C. Decreased C3
D. Shrunken kidney on U/S
E. Increased ASOT
Q-20 – The following drugs can be used in lactation except?
A. Amphetamine
B. Digitalis
C. -
D. -
E. -
Q-21 – Iron poisoning?
A. Vomiting, bloody diarrhea
B. Bleeding gum
C. Tenesmus
D. Convulsion
E. Hallucination
Q-22 – Hypocalcaemia occur in all of the following except?
A. Chronic pancriatitis
B. Pseudo parathyroidism
C. Cushing syndrome
D. CRF
E. -
Q-23 – 3 years child can do the following except?
A. Hopping on one foot
B. Draw a circle
C. Speak in sentences & give full name
D. Building tower of cubes
E. Eat with spoon * folk
Q-24 – Salicylate poisoning ?
A. Antidote should be taken within 16 hrs.
B. If come persist 24 hrs indicate hepatic encephalopathy.
C. If hepatic encephalopathy not occur there is liver cirrhosis
D. --
E. --
Q-25 – Chorio-carcinoma metstatic occur?
A. Liver
B. Lung
C. Brain
D. Ovary
E. Bone
Q-26 – Cyanotic heart disease in the first week with axis deviation?
A. Tricuspid atresia
B. Transportation of great vessels
C. Fallots of tetralogy
D. -
E. -
Q-27 – Cyanotic heart disease associated with decreased pulmonary function?
A. Tricuspid atresia
B. Transportation of great vessels
C. Fallots of tetra logy
D. Complete pulmonary stenosis
E. -
Q-28 – Villous Carcinoma of the vulva simulate clinically?
A. Carbuncle of urethra
B. -
C. -
D. -
E. -
Q-29 – Ovarian tumor common persisting symptom is?
A. Chronic abdominal pain
B. Abdominal distension
C. CVA
D. Pulmonary embolism
E. DIC
Q-30 – Tumor marker is detected in?
A. Mature teratoma
B. Ca cervix
C. Immature teratoma
D. Mature teratoma + Ca cervix
E. Immature teratoma + Ca cervix
Q-31 – Microcytic hypo chromic anemia occur with the following except?
A. Iron deficiency anemia
B. Sideroblastic anemia
C. Scurvy
D. B-thelassemia minor
E. Lead poisoning
Q-32 – Iodine uptake by thyroid at gestation age?
A. 4-6 wks
B. 6-8 wks
C. 8-10 wks
D. 8-12 wks
E. 36 wks
Q-33 – Female with cervical incompetence at 8 wks of pregnancy treatment is?
A. -
B. -
C. -
D. –

Q-34 – Gonadoblastoma is formed by?
A. Germs cells
B. Sex cord stroma
C. -
D. -
E. -
Q-35 – Ovarian tumor with bifid muscles?
A. Brener’s tumor
B. -
C. -
D. -
E. -
Q-36 – Serum amylase increased in all excepts?
A. Acute pancreatic
B. Acute cholecystitis
C. Chronic pancriatitis
D. Acute appendicitis.
E. None
Q-1 – 50 years old female c/o loss of recent preference for cold & palpitation, O/E firm swelling over the anterior neck present for 5 yrs, the most accurate diagnosis is?
A. Simple goiter
B. Diffuse toxic goiter [Grave's disease]
C. Toxic nodular goiter
D. Parathyroid gland adenoma
E. Carotid body tumor
Q-2 – Etiological factor that should be evaluated in a patient with Habitual abortion include all except?
A. Genetic abnormality
B. Rubella infection
C. Uterine abnormality
D. Luteal phase defect
E. Thyroid gland dysfunction
Q-3 – In conscious multiple trauma patient your priority are?
A. To stop bleeding then IV fluid
B. To secure air entry breathing then BP
C. To start an IV fluid & send blood for cross matching
D. To intubate the patient.
E. To do the peritoneal lavage then IV fluid
Q-4 – 36 years old obese man c/o poly urea, weight loss, and urine analysis shows glycosuria, negative for ketone, FBS-280 mg/dl, the most suitable treatment is?
A. Intermittent IM insulin until patient stable.
B. NPH insulin 30 units daily with diabetic diet
C. Sulphonyl urea & diabetic diet.
D. Metaformin.
E. None of the above.
Q-5 – 5 years old boy with leukemia on chemotherapy Becomes febrile to 395 C approximate Response would include all of the following except?
A. Administer aspirin for the fever & observe for temp response
B. Obtained blood culture
C. Obtained chest x-ray
D. Obtained urine culture
E. Start Iv broad spectrum anti-biotic
Q-6 – A 3rd heart sound S3 is typical finding in all of the following except?
A. Tricuspid regurgitation
B. Young healthy athlete
C. Thyrotoxicosis
D. Left ventricular failure
E. Mitral stenosis
Q-7 – 40 years old female with rheumatic heart & mitral valve disease, she has chronic Atrial fibrillation for 6 months, all of the following are indicated except?
A. Electrical cardio version.
B. Warfarin.
C. Digoxin.
D. Echocardiogram.
E. Advised regarding prophylaxis of infective Endocarditis.
Q-8 – 24 years old man has fixation of fractured right femur, 2 days later he becomes dyspnic & had chest pain, Hemoptysis, ABG shows pH-75, PO2-25, which of the following is suitable for management?
A. Frusemide
B. Hydrocortisone
C. Ampicillin
D. Heparin
E. Aminophyline
Q-9 – The following are the sign of CCF in children except?
A. Bounding peripheral pulse
B. Gallop rhythm
C. Basal lung crepitations
D. Hepatomegaly
E. Peri orbital edema
Q-10 – The following are accepted therapeutic option in endometriosis except?
A. Pregnancy
B. Danazol
C. Surgical removal
D. Radiotherapy
E. Progesterone
Q-11 – Cord prolapsed is associated with all of the following condition except?
A. Malpresentation
B. Multiple pregnancy
C. Pre-term premature rupture of membrane
D. Oligo-hydromnios
E. Rupturing of membrane when the head is high
Q-12 – New born infant with abnormal neurological examination it is suspected that he differed Prenatal asphyxia, all of the following complication of pregnancy are risk factors for prenatal asphyxia except?
A. Placental abruption
B. Hyper emesis gravid arum
C. Prematurity
D. Pre-eclampsia
E. Me conium stained amniotic fluid
Q-13 – An infant come with mother to clinic with H/O bleeding per rectum, the most likely diagnosis is?
A. Polyp of he rectum
B. Mickel’s diverticulum's
C. Fissure in ano
D. Intussuption
E. Ulcerative colitis
Q-14 – The following are typically associated with placenta praevia, except?
A. Painless bleeding.
B. Mal presentation.
C. Multiparity.
D. Shock is out of proportion to the amount of bleeding.
E. Non engagement of presenting parts.
Q-15 – Hemorrhagic diathesis following multiple blood transfusions is most likely due to?
A. Platelet depletion
B. Decrease in fibrinogen
C. Decrease in prothrombin
D. Increase in fibrinolytic activity
E. Decrease of calcium
Notes: [complication of massive transfusion is à decrease platelets, calcium, clotting factors, Increase potassium, hypothermia]
Q-16 – 26 years old patient c/o headache, fatigue, Hb-8 gm/dl, MCV-85, retic-10%, of the following investigation the least useful is?
A. Coomb’s test
B. Sickling test
C. Serum bilirubin
D. Serum iron
E. Hb electrophoresis
Q-17 – Biochemical alteration is seen in acute renal failure may include all of the following except?
A. Elevated serum phosphatase
B. Hypourecaemia
C. Hyperkalaemia
D. Elevated serum acid phosphatase
E. Low serum bicarbonate
Q-18 – The following statement about management of UTI in pregnancy are correct except?
A. Asymptomatic bacteriuria should be treated
B. The most common organism is E coli
C. Gentamycin is usually the drug of choice
D. Pyelonephritis can cause septic shock
E. Placenta always should be ruled out
Q-19 – Laparoscopy is indicated in all of the following condition except?
A. Suspected endometriosis
B. Chronic pelvic pain
C. Primary amenorrhea
D. Intestinal obstruction
E. Infertility
Q-20 – DIC may be associated with all of the following except?
A. Pre eclampsia
B. Acute inversion of uterus
C. Abruptio placenta
D. Intra uterine fetal death
E. Diabetes Mellitus.
Q-21 – A young patient come with abdominal pain, the diagnosis of appendicitis is likely if there is?
A. Elevated temperature.
B. Elevated WBC.
C. Rectal bleeding.
D. Tenderness, right lower quadrant with rebound.
E. Urinary symptom.
Q-22 – The following statement about multiple pregnancy are true except?
A. Dizygotic twins are more common than monozygotic twins
B. Twin to twin transfusion is only seen with dizygotic pregnancy
C. Can be diagnosed by U/S
D. Maternal physiology changes are exaggerated compared to single pregnancy
E. Congenital abnormality are more as compared to single pregnancy
Q-23 – Healthy Saudi children routinely immunized against all of the following except?
A. Pertusis
B. Tuberculosis.
C. Mumps.
D. H Influenza type-B
E. Diphtheria.
Q-24 – A 5 years old boy has 1 day cola color urine with RBC cast, two weeks ago positive culture strepto coccus tonsillitis the single best evidence of diagnosed post streptococcal glomerulo-nephritis is?
A. BP above 95 the percentile for ge
B. Positive ASOT
C. Mildly elevated BUN & Creatinine.
D. Negative ANA, HIV & hepatic profile
E. Decreased C3
Q-25 – Complication of obstetric hemorrhage include all of the following except?
A. Irreversible maternal shock
B. Maternal death
C. Fetal death
D. ARF
E. SLE like syndrome
Q-26 – 18 months old child found to have dental decay in the upper central & lateral incisures this is most suggestive of?
A. Excessive fluoride intake
B. Milk bottle cause
C. Tetracycline exposure
D. Insufficient fluoride intake
E. Failure to brush the child teeth properly
Q-27 – Recent weight loss suggests diagnosis of the following except?
A. Thyrotoxicosis
B. Aids
C. Nephrotic syndrome
D. Nephrotic disease
E. Pulmonary TB

Q-28 – All of the following are complication of systemic hypertension except?
A. Intra cerebral hemorrhage
B. Renal artery stenosis
C. Sub dural hemorrhage
D. Lacunars infarcts of internal capsule
E. Left ventricular hypertrophy
Q-29 – 8 years old girls C/O bruises all over the body, leg pain, O/E pallor, ecchymosis & petichiae on the face, WBC-2800/mm3, platelet count 29,000/ mm3, which of the following would be the most suitable diagnostic test?
A. Hb Electrophoresis
B. Bone marrow aspiration
C. ESR
D. Skeletal survey
E. Liver & spleen survey
Q-30 – All of the following condition indicate sever pregnancy include hypertension, except?
A. Pulmonary edema
B. Thrombocytopenia
C. Polyurea
D. Exaggerated reflexes
E. Right upper quadrant pain
Q-31 – Urinalysis may provide useful information about the following renal function except?
A. Renal phosphate handling.
B. Renal protein loss.
C. Renal concentrating capacity.
D. Possible UTI.
E. Possible glomerulo nephritis.
Q-32 – Which of the following is not associated with high amniotic fluid alpha fetoprotein concentration?
A. Anencephaly
B. Meningomyocele
C. Gestroschesis
D. Spinobifida
E. Breech presentation
Q-1 – Secondary hyper lipidemia can be develop due to each of the following except?
A. Hyperthyroidism
B. Nephrotic syndrome
C. Estrogen therapy
D. Alcoholism
E. Hypertension
Q-2 – Which of the following drugs shown to reduce mortality if patient with CCF?
A. Digitalis
B. Frusemide
C. Enalapril
D. Procainamide
E. Aspirin
Q-3 – A 60 years old man came with chest pain of 3 hrs duration associated with ST elevation Lead-II, III & AVF, the heart rate was 46/ min, BP 90/60 mmHg, the initial treatment is?
A. Atropine
B. Isoprenaline
C. Dopamine
D. Temporary pacemaker
E. -
Q-5 - A 48 years old man is admitted to ICU with acute inferior MI after 2 hours BP 86/52 mmHg, heart rate 40/ min with sinus arrhythmia, which of the following is suitable treatment?
A. Immediately insertion of temporary trans venous pacemaker
B. I/V administration of atropine sulphate 06mg
C. Administration of saline
D. I/V administration of sulbutamine 35mg/min
E. I/V administration of isoproternol 50 mg/min
Q-6 – All of the following are true about Grave’s disease, Except?
A. Opthalomo-pathy invariably respond to anti-thyroid therapy.
B. Pre tibial myxedema is a feature.
C. Arterial fibrillation may occur.
D. Patient may have difficulty in climbing stairs.
E. There is often a bruit heard over the thyroid gland.
Q-7 – A 65 years old man presented with difficulty in swallowing food, cause of this condition Include all except?
A. Carcinoma of esophagus.
B. Reflex oesophagitis
C. Cardiac Achalasia
D. Motor neuron disease
E. Herpes oesophagitis
Q-8 - 64 years old man admitted to CCU with anterior wall infarction, on the third day there is Sudden detoriation in patient status, characterized by rapid onset of right & left side carcinogenic Shock, O/E a new harsh systolic murmur is audible, loudest on left sternal border compared by thrill, what is the most likely diagnosis?
A. Papillary muscles dysfunction
B. Pericarditis
C. Rupture of intra-ventricular aneurysm
D. Right ventricular infarction
E. Left ventricular infarction
Q-9 – Acute glucoma, all are true except?
A. It suspected a miotic should be installed prior to refered
B. Headache are typical
C. Has shown to cause abdominal pain
D. Haloes around light & blurred vision uncommonly preced the attack
E. A glucomatous pupil is typically larger then other pupil
Q-10 – The important clinical finding in acute pancriatitis is?
A. Rebound tenderness
B. Presence of some relief on binding forward or maintaining a sitting position
C. Guarding of abdomen
D. Hypoactive bowel sound
E. Abdominal pain
Q-11 – A 24 yrs old man presented with one day history of headache fever, O/E restless, avoid light, neck is persistent to be flexed, which of the following line of treatment is correct?
A. X- ray cervical spine
B. EEG
C. ANA
D. Phenytoid
E. Non of the above
Q-12 – Plasma ferritin is reflection of?
A. RBC mass
B. Total body iron store
C. Hepatic iron content
D. B/M iron store
E. None of the above
Q-13 – A 26 yrs old man presented with headache, fatigue, investigation revealed, Hb- 8 gm//dl, MCV-85, retic 10%, the following investigation are useful except?
A. Coomb’s test
B. Sickling test
C. Serum bilirubin
D. Serum iron
E. Hb electrophoresis
Q-14 – The following may induce seizure except?
A. Hypoxia
B. Hypourecaemia
C. Hypokalaemia
D. Hyponatremia
E. Hypocalcaemia
Q-15 – The following diseases are recognized causes of high output cardiac failure except?
A. Paget’s disease
B. Thyrotoxicosis
C. Hypertension
D. Arterio venous fistula
E. Iron deficiency anemia

Note: cardio myopathy, ischemia, MI, as low out put failure.

Q-16 – A 30 yrs old man c/o generalized fatigue ability for 2 months, O/E generalized lymph adenopathy,
D/D of this case include all of the following except?
A. Lymphoma
B. HIV infection
C. HCV
D. Infectious mononucleosis
E. Toxoplasmosis
Q-17 – Obstructed jaundice is associated with all except?
A. Elevated indirect serum bilirubin
B. Elevated direct serum bilirubin
C. Uro bilinogenaemia is almost always present
D. Presence of bilirubinaemia
E. Elevated alkaline phosphatase
Q-18 – In RA, all are true except?
A. Causes destruction of articular cartilage
B. Is frequently associated with HLA-DR4
C. Is equally common in males & females
D. Is characterized by the presence of nodules
E. Can involved any synovial joint in the body
Q-19 – Which of the following indicate pre-renal failure?
A. Casts in the urine
B. Low urine osmolality [<400 mmmol/l]
C. Low urine sodium concentration[<20mmol//l]
D. Low free water excretion
E. Microscopic hematuria
Q-20 – Hyponatremia occur in the following condition except?
A. Primary adrenocortical insufficiency
B. CCF
C. Diabetic ketoacidosis with very high blood sugar
D. Excessive production of vasopressin
E. Diabetes insipidus
Q-21 – All of the following are causes of hypocalcaemia except?
A. Primary hyperthyroidism
B. Familial benign hyper calcaemia
C. Malignancy
D. Cushing syndrome
E. Granulomatous disorder [sarcoidosis]
Q-22 – About Germen measles [Rubella]?
A. Has an incubation period of 3-5 days
B. Usually begins with high fever
C. Its cause arthritis
D. Cause of oral ulceration
E. Does not cause cardiac abnormality & deafness in fetus
Q-23 – Viral hepatitis?
A. In most of the cases due to HbsAg
B. Is more likely to lead to chronic hepatitis after recovery from an acute fulminating attack than
C. After a milder illness when associated with cholelithiasis & pruritis carries a bad prognosis.
D. HbsAg is found in about 20% pt with Polyarthritis nodosa
E. Its always associated with joint pain in the prodromal phase
Q-24 – Each of the following statement about HbsAg is true except
A. In HbsAg +ive mother there is very high probability of transmission to new born.
B. Antenatal screening for HbsAg viru should be carried out only in high risk group
C. Less common risk factor for hepatitis B virus include multiple episodes of veneral disease & as a blood donor
D. Injection of hepatitis-B immunoglobin is indicated in person exposed to accidental pin prick of positive patient.
Q-25 – About allopurinol [zyloric] all are true except?
Its of value in acute phase.
A. Is uricosuric.
B. Is contraindicated in patient with renal failure.
C. Action is antagonized by salicyalate.
D. Reduce the risk of uric acid stone
Q-26 – About skin pigmentation all are true except?
Pregnancy
A. Addison’s disease
B. Hemochromatosis
C. Neurofibromatosis
D. Cirrhosis
E. Chronic arsenic poisoning
Q-27 – Essential diagnosis of Psoriasis all are true except?
A. Silvery scales on bright red, well demarcated plaque, usually on the knee, elbow & scalp
B. Nail finding including pitting & oncholysis[separation of the nail plate from the bed]
C. Mild itching usually
D. May be associated with psoriatic arthritis
E. Histopathology is diagnostic
Q-28 – D / D of acute appendicitis include the following except?
A. Mittle schinerz disease
B. Carcinoma of caecum
C. Pelvic inflammatory disease
D. Diverticulitis
E. Mesenteric lymphadenitis
Q-29 – The greatest metabolic activity of the thyroid hormone due to?
A. Thyroxine
B. Mon iodotyrosine
C. Di iodotyrosine
D. Tri iodothyroxine [T3]
E. Thyroglobulin
Q-30 – Complication of osteomyelitis include the following except?
A. Septic arthritis
B. Arrest of bone growth
C. Pyomyositis
D. Septicemia
E. Destruction of epiphyseal plate
Q-31– Which of the following congenital heart disease is associated with lowest risk of infective endocarditis?
A. PDA
B. Tetralogy of fallot
C. Sub valvular aortic stenosis
D. ASD
E. VSD
Q-32 – All of the following statement regarding the association [formerly] campylobacter pylori and gastritis are true except?
A. Growth of H pylori in the stomach is believed to be a cause of chronic gastritis where as growth in the duodenum is not clear.
B. H pylori is a gram negative bacilli that invade gastric mucosa, there by producing mucosal inflammation
C. H Pylori can be identified by its ability to clear urea.
D. Eradication of the bacteria result in histological improvement of mucosa..
E. Ampicillin or metronidazole can be eradicate H pylori
Q-33 – The single feature bets distinguish crohn’s disease from ulcerative colitis is?
A. Presence of ileac disease.
B. Cigarette smoking history.
C. Presence of disease in the rectum.
D. Non caseating granuloma.
E. Crypt abscess.
Q-34 – Blistering bulae rashes occurs in the following except?
A. Erythema multiform
B. Barbiturate over dose
C. Dermatitis herpetiforms
D. Erythema nod sum
E. Allergy to sulphonamides
Q-35 – Each of the following simulate acute abdomen except??
A. Tonsillitis
B. Pleurisy
C. Polyarthritis nodosa
D. Herpes zoster
E. Myocardial infarction
Q-36 – Each of the following statement about spontaneous abortion is true except?
A. The risk of spontaneous abortion increased with maternal age
B. Sensitive pregnancy test may remain positive after 2 weeks of fetal death
C. Death of embryo usually proceeds vaginal bleeding in early pregnancy loss.
D. A sever co-agulation disorder complicating a missed abortion will usually occur within fewer weeks of fetal death.
E. None of the above.
Q-37 – Correct statement concerning Wilm’s tumor include all of the following except?
A. The average age at diagnosis is 1-3 years
B. The presenting symptom is an abdominal mass
C. Pulmonary metastasis is common
D. The cure rate in the absence of metastasis is high
E. The cure rate in the presence of metastasis is low.
Q-38 – Each of the following statement about post pills amenorrhea is true except??
A. It is more common in women with history of menstrual irregularity.
B. The incidence rises with increased duration of use of oral contraceptive
C. Full investigation is necessary if it persist more than 6 months
D. Pregnancy should be considered in differential diagnosis.
E. Non of the above
Q-39 – Aluminum & calcium salt inhibit the intestinal absorption of which of the following agent?
A. Isoniazide
B. Chloramphinicol
C. Phenoxy methyl penicillin
D. Erythrocin
E. Tetracycline
Q-40 – A young man taken refrigerator food, 3 hours later he develop watery diarrhea, which of the following is the cause?
A. Shigella
B. Salmonella
C. Compylobacter
D. Staphlococcus
E. Giardia
Q-41 – In mitral stenosis the following may be seen except?
A. P neutral on ECG
B. Double contour of right border of heart [due to leftatrial enlargement]
C. LVF
D. RVF
E. Atrial fibrilation
Q-42 – Giardiasis?
A. Involve mainly the terminal ileum & colon
B. It is more common in hypo gamma globinaemic patient
C. Liver abscess is a recognized complication
D. It is best treated by mebendazole
E. May cause lower GIT bleeding
Q-43 – The commonest cause of acute pancriatitis in adult in our country is?
A. Unknown
B. Parasite
C. Mumps virus
D. Biliary stone
E. Ethanol
Q-44 – 16 years old women pregnant is at high risk for the following problems except?
A. Premature deliveries
B. Pelvic complication
C. Toxemia
D. Low birth weight
E. Increased prenatal death rate
Q-45 – Uncomplicated seasonal allergic rhinitis is associated with the following except?
A. Nasal & palatal pruritis.
B. Thin watery nasal discharge.
C. Redness nasal mucosa & cervical adenopathy.
D. Paroxysmal sneezing.
E. Excess lacrimation.
Q-46 – Treatment of all the family is required when the following disease is detected in one of the family members?
A. Malaria.
B. Schistosomiasis.
C. Hook worm infestation.
D. Scabies.
E. All of the above.


Q-47 – A 2 months old child was found to be jaundiced to have an enlarge cirrhotic liver, ascities, diagnosis is?
A. Gilbert’s syndrome
B. Crigler Najjar syndrome
C. Dubin Johnson syndrome
D. Congenital billiary atresia
E. Sickle cell disease
Q-1 – Typhoid fever can be characterized by all of the following except?
A. The illness usually acquired from ingestion of contaminated food, water or milk.
B. Leucopenia is more common than leucocytosis in acutely ill pt
C. Rose spot are usually present at the time of fever start
D. Chloramphinicol is not effective in preventing relapse
E. Quinolone antibiotics eradicate the organism even in the presence of gall stones
Q-2 – 40 years old Phillipino man has hypo pigmented macular lesion & palpably enlarged lunar nerve, the diagnosis of leprosy can be best established by?
A. Positive lepromin test
B. Culture of organism on material obtained by skin biopsy
C. Development of Erythema & swelling of the lesion after a trial of dapsone therapy
D. Demonstration of acid fast organism in skin lesion
E. None of the above, leprosy is a clinical diagnosis.
Q-3 – Which of the following is a usual feature of interstitial lung disease like fibrosing alveolitis?
A. Fever
B. Hemoptysis
C. Generalized wheeze
D. Purulent sputum
E. Fine crepitations
Q-4 – What is the drug of choice in exercise induce asthma
A. Theophyline
B. Atropine
C. Isoproternol
D. Cotrimaxazole
E. Prednisolone
Q-5 – 18 months old boy, his mother concerned about speech, he say mama & baba only, he is healthy since birth, evaluation of this child should begin with?
A. Physical examination
B. Chromosome examination
C. Hearing evaluation
D. Development testing
E. CT scan of the head
Q-6 – Which of the following diseases is usually increased maternal mortality when occur in pregnancy?
A. Syphilis
B. Biliary cholestasis
C. Phechromocytoma
D. Herpes genitalis
E. Toxoplasmosis
Q-7 – Baby born at term, immediately after delivery he develop sever respiratory failure, breath sound decreased bilaterally, abdomen is flat, X-ray chest multiple cystic masses in the left side of chest, mediastenum shift to right, its likely syndrome is?
A. Respiratory distress syndrome
B. Diaphragmatic hernia
C. Congenital lobar emphysema
D. Persistence fetal circulation
E. -
Q-8 – Concerning obstructed labor?
A. More common in primigravida
B. Excessive caput & molding are recognized sign
C. Its common in occipito-anterior position
D. Oxytocin is indicated in its management
E. Can be diagnosed early before onset of labor.
Q-9 – 15 years old boy, diagnosed partial thickness burn of skin, which of the following statement is true about the burn?
A. Its sensitive
B. Its insensitive
C. Its required split skin graft
D. It will separate as a slough in 2-3 wks
E. Its require a free flap graft
Q-10 – 30 years old man c/o left side weakness 3 hrs duration, o/e pulse-90/min irregular, BP-120/70 mmHg, diastolic murmur in mitral area, on left side hemiperesis, the most likely treatment is?
A. Heparin
B. Digoxin
C. Lumber puncture
D. EEG
E. Cardio angiography
Q-11 – Which of the following is the best for Acute Gouty arthritis?
A. Allopurinol
B. Penicillinoid
C. Indomethacin
D. Gold salt
E. Paracetamol
Q-12 – About Infertility?
A. Is failure to conceive after 6m of trail
B. Is due to an ovulation in minority of case
C. Can be due to high Prolactin level
D. Due to F tube problem, can only be diagnosed by histo-salphingiography.
E. Is due to male factor in majority of case
Q-13 – 5 years old girl presented with 5 days fever and chills, P/F positive for MP, in which of the Following statement is correct?
A. Banana shaped gametocytes present in P Vivax
B. Treatment should be started immediately with Primaquine 10 mg/kg for 3 days
C. Thrombocytopenia is not a feature of Malaria & should make you suspect another diagnosis.
D. P Falciparum is the most likely malaria species
E. Respond to anti-malarial therapy usually tale 72 hrs or more.
Q-14 – Secondary amenorrhea?
A. Is rarely due to an ovulation
B. Can be due to gonadal agenesis
C. Is part of Sheehan's syndrome
D. Its commonly due to Turner's syndrome
E. Its always pathological
Q-15 – Regarding to breach presentation, the following are true except?
A. The incidence after 36 wks is 22%
B. Intracranial hemorrhage is known complication of delivery
C. Is some time associated with congenital uterine anomalies
D. Prematurity is known predisposing factor
E. Its an abnormal presentation
Q-16 – All of the following are useful in the therapy of unstable angina pectoris, except?
A. Heparin drip
B. Aspirin
C. IV nitroglyceride
D. Beta blocker therapy
E. Enalapril
Q-17 – In childhood TB, important diagnostic points include all of the following, except?
A. A history of contact with the disease
B. A positive tuberculin test
C. AFB positive in gastric levage
D. Abnormal X-ray chest
E. significant spleen enlargement


Q-18 – The first drug to be used after commencing cardio-pulmonary resuscitation in a child With?
A. Atropine
B. Adrenaline
C. Lidocane
D. Na-barbiturate
E. Ca-chloride
Q-19 – Fibroid & pregnancy?
A. Can leads to sever anemia
B. Commonly leads to anti-partum hemorrhage
C. Should be remove surgically
D. Is most likely to regress after delivery
E. Is commonly complicated by torsion
Q-20 – Which of the following polyp's is generally considered pre malignant?
A. Pedunculated polyp
B. Villous adenoma
C. Polypoid adenoma
D. Juvenile polyp
E. Pseudo polyp
Q-21 – 6 months old lethargic infant is brought to emergency, with temp- 39 C, which organism Can cause sepsis in the child??
A. Staphylococcus epidermidis
B. Listeria monocytogenous
C. Staphylococcus aureus
D. Streptococcus group-B
E. Hemophilus influenza typ-B
Q-22 – In congenital dislocation of hip joint, the following statement is true?
A. The diagnosis is establish at 3 yrs of age
B. Abducting the flexed hip can cause click[orlanis sign]
C. Abducting the flexed hip is not limited
D. Apparent lengthening of the thigh with the hips & knee flexed may be seen
E. Initial treatment is by open reduction
Q-23 – In portal hypertension, the associated hepato-cellular damage is least if the cause is Iron over load?
A. Schistosoma mansoni
B. Post necrotic scarring
C. Alcoholic liver disease
D. Cirrhosis following chronic active hepatitis
E. -
Q-24 – The group A beta hemolytic streptococcus may trigger the attach of acute Rheumatic Fever, when it ?
A. Spread via blood
B. Causes an upper respiratory tract infection
C. Lodge in the myocardium
D. Invade the joints
E. Enter through a skin infection.
Q-25 – 6 years old girl, low grade fever, arthralgia 5 days, difficulty in swallowing associated with fever 3wks prior to presentation, OE, heart rates 150/min, pan systolic murmur at apex, no gallop, liver 1 cm below the costal margin, the most likely diagnosis is?
A. Bacterial Endocarditis
B. Viral myocarditis
C. Acute Rheumatic fever
D. Peri carditis
E. CCF
Q-26 – 24 years old man, car accident, fracture pelvis, bleeding urethra, you should?
A. Put Foley's catheter to drain any blood in the UB.
B. Stabilized the pelvis & suprapubic catheter
C. Stabilized the pelvis & Foley's catheter
D. Do IVP if normal put FC
E. Put FC & suprapubic catheter
Q-27 – 35 years old female H/O PPH at age of 22yr, C/O amenorrhea, loss of axillaries hair & pelvic hair, atrophy of external genitalia, all of the following is true, Except?
A. X-ray skull showing ballooning of the sella
B. Hyponatremia
C. Hypoglycemia
D. Low serum T4
E. Low radio active iodine uptake
Q-28 – Peripheral neuritis can be caused by the following except?
A. DM
B. Lead poisoning
C. Gentamycin
D. Uremia
E. -
Q-29 – 35 years old female H/O bright red blood discharge from nipple, O/E, cystic swelling felt beneath the areola, the diagnosis is?
A. Breast abscess
B. Fibro adenoma.
C. Duct papilloma
D. Duct ectasia
E. Fat necrosis of the breast

Q-30 – The child who is poisoned, the most effective treatment to remove gastric contant Is by use of?
A. Saline lavage with wide bore nasogastric tube
B. Tartar emetic
C. Syrup of ipeca cuanah
D. Manual induction of vomiting
E. Citrate of magnesia
Q-31 – 30 years old male with long H/O Cronh’s disease, surgery is indicated if he has?
A. Internal fistula
B. External fistula
C. Intestinal obstruction
D. Abdominal mass
E. Stagnant bowel syndrome
Q-32 – All of the following are S/S of normal pregnancy except?
A. Hyper emesis
B. Amenorrhea
C. Hagar's sign
D. Chadwick sign
E. Quickening
Q-33 – 32 years old pt H/O low back pain & morning stiffness, O/E tenderness over sacroiliac Joint, loss of lumber lordosis, x-ray shows sclerosis of sacro-iliac joint, all of them are true About disease, except?
A. The disease is more common in male than female [9:1]
B. Arthritis is more common complication
C. RA factor is negative
D. Subcutaneous nodules are frequent
E. Phenylbutazone + indomethacin are effective therapy
Q-34 – 50 years old man came with rectal bleeding, O/E external hemorrhoid, you will do?
A. Advised excision of hemorrhoid
B. Do barium enema or colonoscopy
C. Give iron tab
D. Do rigid sigmoidoscopy & if normal excise hemorrhoid
E. Do nothing & ask pt to come after 6 months
Q-35 – Primary amenorrhea may be due to?
A. Failure canalization of the mullerian duct
B. Congenital adrenal virilism
C. Kallman’s syndrome
D. All of the above
E. Non of the above
Q-36 – The anatomical site of the neck of femoral hernia is?
A. Transverse fascia
B. Ileopectineal ligament
C. Femoral ring
D. Cribriform facia
E. Obturator foramen
Q-37 – 40 years old man, H/O gall stone, CO sever abdominal pain, O/E ecchymotic discloration of the flanks, the most likely diagnosis is?
A. Acute colitis
B. Acute cholecystitis
C. Acute appendicitis
D. Acute pancriatitis
E. Perforated duodenal ulcer
Q-38 – All of the following about moving eyelocelle are true, except?
A. Usually associated with hydrocephalus
B. Its fatal if not treated in 24 hrs
C. It may be diagnosed in utero with ultrasound.
D. It may cause urological problem.
E. It's require orthopedic management.
Q-39 – The best predictive sign of severity of an asthmatic attack is?
A. Intercostals & supra clavicular retraction
B. Paradoxical pulse of 20 mmHg
C. Arterial PO2 of 50 mmHg
D. Arterial PO2 of 60 mmHg & PCO2 of 44 mmHg
E. Visible exhaustion

Q-40 – All of the following statement concerning lactation are true, except?
A. Prolactin is essential for lactation
B. Sucking trigger a rise in Prolactin
C. Sucking stimulate the release of Oxytocin
D. Expression of milk is due to contraction of myo-epithelial cells
E. Milk let down may be inhibited by over hydration.

SAUDI COUNCIL FOR HEALTH SPECIALIST EXAM
Sample-1
Q-1 – In abdominal trauma, all true except?
a- spleen is the common damage organ
b- badly injured spleen needs spleenectomy
c- Abdominal lavage often excludes abdominal hemorrhage.
d- Abdominal often accurate to localize the site of trauma.
e-
Q-2 – Ano-rectal abscess, all true except?
a- First line of treatment ABC
b- Physical signs can be hidden if it is in supra-levator space
c- Usually originates from intra-sphinctric space.
d- Usually originated from anal gland infection.
e-
Q-3 – Intestinal obstruction, all true except?
a- Increased temperature & pulse with localized rigidity & tenderness indicate strangulation.
b- Serum amylase could be elevated..
c- Always require surgery..
d- If high obstruction the distension will be absent,
e-
Q-4 – Acute appendicitis in children?
a- Leucocytosis is diagnosis.
b- Rarely perforated if untreated well
c- Can cause intestinal obstruction.
d- Need ABC before surgery for every child
e-
Q-5 – In breast Carcinoma all of the following are true, except?
a- 2 cm mass with free axilla in stage-1.
b- Chemotherapy is must for pre-menopausal with positive axilla
c- Radical mastectomy is the choice of surgery.
d- Mammography per year for contra lateral breast.
e-
Q-6 – Inhalation injury in burned, all of the following are true, except?
a- CO is the major cause of death in early stage.
b- Patient should be admitted to ICU for observation even without skin burn
c- Signed vibrissae is respiratory sign
d- Bronchioles & alveoli could burn from hot smoke.
e-
Q-7-Ischemic leg?
A. Golden period 4-16 hours
B. nerves first to be damage.
C. angiogram done in all Pt.
D. Parasthesia pt more critical than those with pain
E. -
Q-8-Acute cholangitis, all of the following are true ECEPT?
A. coli most common.
B. septicemic shock most likely complication
C. Jaundice uncommon.
D. ERCP & papillotomy is best treatment
E. -
Q-9-Right colon Ca, all of the following are true ECEPT?
a- Profound anemia.
b- Occult blood
c- Dyspepsic symptoms
d- Melena,
e-

Q-10-Recent hemothorax
a-thoraccotomy & decortication
b- Aspiration
c- Insertion of chest tube
d- Volume replacement only.
e-
Q-11- Thyroid Ca associated with?
a- hyperthyroidism
b- hypothyroidism
c- euthyroid
d- only metastatic tissue produce hormones
e- toxic nodules.
Q-12- Fractured humorous associated with most common?
a- radial nerve injury
b- ulner nerve injury
c- medial nerve injury.
d- axillary nerve injury
e- musclocutaneous nerve injury
Q-13- Fractured pelvis most commonly associated with?
a- bladder injury.
b- penile urethra injury.
c- bulbomembranous urethra injury
d- ureter injury
e-
Q-14- Best treatment for tension pneumothorax “patient in distress”?
a- I.V. Fluid.
b- Oxygen
c- respiratory sti,ulator.
d- Aspiration of ait by needle.
e- Intubation.
15- Ranulla is?
a- forked uvula.
b- Thyroglossal cyst
c- Swelling at the floor of the mouth.
d-
e-
Q-16-Critical count of platelets which leads to spontaneous bleeding is?
a- 1000
b- 50,000
c- 75,000
d- 100,000
e- 200,000
Q-17- which one will give bilateral breast Ca?
a- lobular breast Ca.
b- Intraductal Breast Ca
c- mucinous breast Ca
d- Medullary breast Ca.
e- Tubular breast Ca.
Q-18- The best method for temporary control of bleeding id?
a- Arterial tourniquet.
b- Venous torniquet.
c- Direct venous pressure.
d- Adrenaline
e-
Q-19- Anorexia nervosa, all of the following are true ECEPT?
a- Lethargy.
b- Lanugo hair
c- Amenorrhea
d- Young female
e-
Q-20- Hypochondriasis, all of the following are true ECEPT?
a- More common in medical student.
b- less commoc in male than females
c- more common in lower social class.
d- defined as morbid pre-occupation of one’s body or health.
e-
Q-21- All are speech disorder, Except?
a- Stuttering.
b- Mumbing
c- Cluttering
d- Paliala.
e-
Q-22- Family behavior towards schizophrenic pt affect prognosis adversly?
a- Double binding
b- Over emotion behavior
c- Schizomatic parents
d- Projective identification
e-
Q-23- Pre-ejaculation, all of the following are true EXCEPT?
a- Most common sexual disorder in males.
b- Uncommon in young men.
c- Its treatment benefit from other partner.
d- it benefit anxiety treatment.
e-
Q-24- The most prominent symptom of acute otitis media is?
a- Pain.
b- Hearing loss.
c- discharge..
d- Tinitis
e- Non of the above.
Q-25- Lenth of the trachea in adult is?
a-11-12 cm.
b- 24 cm
c- 20 cm.
d- 4 cm
e- non of the above
Q-26- Identification for trachestomy, all of the following are true EXCEPT?
a- Foreign body in larynx.
b- left recurrent nerve cut
c- Ca larynx
d- In some procedure exposed to radiation
e- non of the above
Q-27- Fetal unilateral discharge is feature of?
a- adenoid
b- choanal atresia.
c- Foreign body..
d- right atrophy..
e- non of the above.
Q-28- Best first aid to control epistaxis is?
a- Adrenaline
b- Cold application on forehead
c- Good pinching or compression lower end of nose for 5-8 minutes.
d- non of the above
e-
Q-29- Known risk factor of suicide include all of the following are true EXCEPT?
a- Depression
b- Previous self attempt
c- Females less than males.
d- Drugs & alcohol dependence,
e- If doctor ask the pt any suicidal attempt..
Q-30- Gastric lavage?
a- Inffective after 12 hrs paracetamol intake..
b- indicated with paraffin oil.
c- used more in semiconcious pt than induced vomitting
d- pt sfould be on right side.
e-
Q-31- A 50 years old man presented with central abdominal pain radiated to the back, X-ray abdomen normal, X-ray back normal, diagnosis is?
a- Cholecystitis…
b- Appendicitis.
c- Pancreatitis.
d- Diverticulitis.
e-
Q-32- Acute glomerulonephritis, which is not correct investigation?
a- Complement.
b- Urinanalysis.
c- ANA
d- Blood culture.
e- Cystoscopy.
Q-33- all of the following precipitate seizure EXCEPT?
a- Hypourecemia.
b- Hypokalemia.
c- Hypophosphatemia..
d- Hypocalcemia.
e-
Q-34- A 25 yrs pt presented with headache, avoidance of light & resist flexion of neck, next step is?
a- ECG
b- X-ray cervical spine..
c- Phenytoin
d- non of the above.
e-
Q-35- A 20 yrs old female present with fever, loin pain & dysuria, management all of the following EXCEPT?
a- Urinalysis & culture.
b- Blood Culture.
c- IVP
d- Cotrimexazol.
e- non of the above.
Q-36- The most effective monitor in pt with acute bleeding?
a- Hb.
b- HCV
c- vital sign
d- Amount of blood loss..
e-
Q-37- the most common cause of epistaxis in children is?
a- Polyps.
b- trauma.
c- Dry air.
d- Thrombocytopenia
e-
Q-38- The amount of Na, in ORS in WHO is?
a- 150 meq
b- 120 meq
c- 90 meq
d- 60 meq
e- 30 meq
39- Child with epiglotitis with present with all of the following EXCEPT?
a- Fever
b- Dysphagia
c- like to lie in supine Position
d- Stridor
Q-40- The Likelihood Of a daughter For Father having severe hemophilia B is:
a- 0%
b- 25 %
c- 50 %
d- 75 %
e- 100 %
Q-41- All Of the following are true about pyloric Stenosis, EXCEPT:
a- Incidence male more than female.
b- Onset is generallt late in the first month of life.
c- Vomitus is bile stained.
d- Appetite is good.
e- Jaundice occur in association.
Q-42- Risk factor of sudden death syndrom include all of the following, Except?
a- Cigratte smoking during pregnancy.
b- Old primigravida.
c- Crowded living room.
d- Prematurity.
e- small gestational age.
Q-43- A 2 yr boy has rectal pain, bleeding with perianal itching & constipation for 3 days, physical examination revealed a perianal erythematous rash which extend 2 cm around the anal ring, most likely diagnosis is?
a- Anal fissure.
b- Rectal polyp.
c- Ulcerative colitis.
d- Streptococcal infection.
e- Malakoplakia.
Q-44- In brucellosis, all of the following are true EXCEPT?
a- Brucella. A cause more sever form than B.M in children..
b- Human to human rarely documented.
c- Human can infected through inhalation.
d- Brucella species are small. Non-motile gm -ive coccobacilli.
e- pt with higher titer can show false -ive..
Q-45- Children are expected to walk without support at age of?
a- 6 m
b- 9 m
c- 15 m
d- 18 m
e- 20 m
Q-46- Which of the following vaccines not given to a household contact with immunodeficient child?
a- Hepatitis.
b- DPT
c- Oral polio
d- BCG
e-
Q-47- Sympotm of cystic fibrosis in neonates?
a- Meconium ileus..
b- Pneumothorax
c- Steatorrhea.
d- Rectal prolapse.
e-
Q-48- DKA in children, all of the following are true EXCEPT?
a- Don’t give K till lab result come.
b- ECG monitering essential..
c- If pH < 7..0 than give HCO3.
d- NGT for semiconscious pt.
e- Furosemide for pt with oliguria..
Q-49- Common sypmtom of Hodgkin lymphoma not seen in Non Hodgkin L?
a- Night sweat
b- Superior vena cava syndrome
c- CNS involvement.
d- Intussusception
e- Bone pain.
Q-50- To prevent tetanus in neonates
a- Give antitetanus serum in neonates.
b- Give immunoglobinto mother..
c- Give tetanus toxoid to mother
d- Give antibiotic to mother
e- give penecillin to child to kill tetanus bacilli.
Q-51- MMR given at age of
a- 3m
b- 8 m
c- 12 m
d- 24m
e- 30m
Q-52- Hypothyroid in young baby usually due to?
a- Endocrine irresponse.
b- enzyme deffiency.
c- Drug use by mother..
d- Agenesis
e-
Q-53- Blood pressure, all of the following are true EXCEPT?
a- If 2/3 of cuff à false increased BP.
b- internal cuff must cover 80% of arm
c- following circadian vary à increased late night BP.
d- Increased BP à 3 standrad deviation away from normal.
e- You have to have more than one reading to diagnosis inreased BP.
Q-54- All of the following drugs advised NOT to be given to elderly patien except?
a- Cimitidine.
b- Thyroxin
c- Digoxin
d- Chloropromide
e-
Q-55- Percentage of reinfection for pt undergoing non-cardiac surgery
a- 5% in 3 months.
b- 15% in 3 months
c- 35% in 3 months
d-5% in 6 months
e- 35%IN 6 MONTHS
Q-56- Fruosemide increased excretion of?
a- Na
b- K
c- Phosphorus
d- Non of the above
e-
Q-57- Heparin action depend on?
a- Potetiation of antithrombin III
b- Change plasmin to plasmogen
c- Affect prothrombin
d- affect ionized Ca
e-
Q-58- Digoxin toxicity?
a- Tinitis
b- Pleural effusion
c- nausea
d- All of the above
e- non of the above
Q-59- Communicable disease controlled by?
a- Control of the sorce of infection
b- Block the causal of transmission
c- Protect the susceptible pt
d- All of the above
e- non of the above.
Q-60- Anti-D Ig not given to a pregnant if?
a- 25-28 wks
b- Anti-D antibody ratio 1:8
c- after amniocentesis
d- after anti partum hemorrhage
e- after chrionic villi biopsy
Q-61- blood pH?
a- Incread after diarrhea
b- decreased after vomitting
c- more in right atrium than left atrium
d- lower in right atrium than left atrium
e- lower in renal vein than renal artery

Q-62- Premenstrual tension
a- more in first half of menses
b- 60% associated with edema
c- associated with increased eating salty food
d- menorragia
e-
Q-63- blockage of first stage labor pain by?
a- Block of lumbosacral plexus afferent?
b- Block of lumbosacral plexus efferent
c- Block of pudendal nerve.
d- Block of sacral plexus.
e-
Q-64- If a pregnant women eating well balanced diet, one of the following should be supplied?
a- Ca.
b- Phosphorus.
c- Vit-C
d- Non of the above
e-
Q-65- Most important cause of immediate post partum hemorrahage?
a- Laceration of cervix.
b- Laceration of vagina
c- Uterine atony.
d- Placental fragment retension
e-
Q-66- Dysparunea caused by all of the following Except?
a- Cervicitis.
b- Endometritis
c- Lack of lubricant
d- Vaginitis
e- Uterine prolapse
Q-67- all of the following are normal flora & should not treated Excpet?
a- Trichomonas.
b- Candida
c- E coli
d- Fragmented bacteria
e-
Q-68- Treatment of bacterial vaginitis?
a- Ampicillin.
b- Tetracyline
c- Metronidazole.
d- erythromycin
e-
Q-69- Management of anaphylactic shock, all of the following Except?
a- I.V.F
b- 100% oxygen
c- corticosteroid
d-
e-
Q-70- all of the following are sign of allergy to local anesthesia, Except?
a- Laryngeal spasm
b- urticareia.
c- decreased B.P
d- Bronchospasm
e-
Q-71- Gastric aspiration?
a- Cuffed NGT may prevent aspiration.
b-
c-
d-
e-

Q-72- Below the inguinal ligament, where is the femoral artery?
a- Medial
b- Lateral
c- Anterior
d- Posterior
e-
Q-73- Hepatitis most commonly transferred by blood?
a- HBV
b- HAV
c- Non-A, Non-B
d- Non of the above.
e-
Q-74- Primary Tb?
a- Upper part of lung..
b- Normal X-ray
c-Positive PPD
d-
e-
Q-75- Increased bleeding time, all of the following Except?
a- hemophilia
b- scurvy
c-
d-
e-
Q-76-Serum ferritin reflect?
a- total iron store
b- Serum iron
c- Bone morrow iron.
d- Non of the above.
e-
Q-77- which one shift oxygen dissociation curve to the left?
a- Hypoxia.
b- Acidosis
c- High attitude
d-
e-
Q-78- Treatment contact, all of the following Except?
a- Belariasis
b- Malaria
c- Hook worm
d-
e-
Q-79-The best way of health education?
a- Mass media.
b- interview
c-
d-
e-
Q-80- Which one cab ne taken orally?
a- Amikacin.
b- Neomycin.
c- gentamycin.
d-
e-
Q-81- Chronic use of steroid will give?
a- Osteomalcia
b- Myopathies of pelvic girdle muscles.
c- Increased risk of breast Ca..
d-
e-

Q-82-Swallowed foreign body will be found in, all of the following Except?
a- Stomach
b- Tonsil
c- Pharyngeal pouch.
d-
e-
Q-83- all of the following are true about pulmonary embolism, Except?
a- Normal ABG
b-
c-
d-
e-
Q-84- all of the following cause gastric irritaion, Except?
a- Erythromycin
b- NSAID
c- Sucralfate
d- declovenac
e-
Q-85- all of the following are anti-arrhythmic drugs, Except?
a- Xylocane
b- Digoxin
c-
d-
e-
Q-86- APGAR score?
a- Heart rate important.
b- of 12 point.
c-
d-
e-

Q-87- Old male pt came with urine retention, dilated ureter & hydronephrosis, dignosis is?
a- Benign prostate hypertrophy.
b-
c-
d-
e-
Q-88-in DKA use?
a- Short & intermediate insulin..
b-
c-
d-
e-
Q-89- Torsion, all of the following are true, Except?
a- Very tender & progressive swelling.
b- More common in young males
c- hematuria
d- treatment is surgical
e-
Q-90- Congenital squint ?
a-
b-
c-
d-
e-
Q-91- all of the following cause secondary HTN, Except?
a- Pheochromocytoma.
b- Addisons disease.
c- Hyperaldosterone.
d- Renal disease.
e-
Q-92- all of the following are true about paracetamol poisoning, Except?
A. Metaboilc acidosis.
B. Hypoglycemia.
C. Brochospam
D. Liver failure
Q-93- Adenoid
A. Can be chronic source of infection.
B. Cause snoring by movement
C. Attached to roof of nose
D. Involved in immune system reaction
Q-94- Cellulitis in children [6-24 m] mostly cause by?
A. H. Influenza..
B. Streptococcus group-A
C. Staphlococcus.
D. Psuedomonas
E. e. coli
Sample-2
Q-95- all of the following drugs should ve avoided during pregnancy Except?
A. Na Valporate
B. Glibinclamide
C. Ceflex
D. Septran
E. Warfarin
Q-96- Secondary prevention is best effective in?
A. DM
B. Leukemia
C. Pre-eclemsia
D. Malabsorption
E. -
Q-97-Complication of colostomy, all of the following Except?
A. Malabsorption of water
B. Prolapse
C. Retraction
D. Obstruction
E. Excoriation of skin
Q-98- Rubella infection ,one is TRUE:
a- Incubation period is 3-5 days
b- Oral ulcer
c- Arthritis
d- Not cause heart compliction to the fetus
e-
Q-99- Avascular necrosis of head of femur usually detected clinically by
a-3 months
b- 6 months
c- 11 months
d- 15 months
e-
Q-100- All of the following may cause sudden unilateral blindness, EXCEPT:
a- Retinitis pigmentosa
b- Retrobulbar neuritis
c- Retinal detachment
d- Veterus hemorrhage
e- Central retinal artery embolism
Q-101- Glue ear, one is TRUE
a- can be treated by Grumett tubr insertion
b-
c-
d-
e-
Q-102- the First symptom of left heart failure is
a- Orthopnea
b- Edema
c- Dyspnea on exertion
d-
e-

Q-103- Tinnitis one is TRUE
a- Not expert by children
b- iii
c- iii
Q-104- Treatment of tetralogy of fallot, all of the following true Except?
A. Thoracotomy
B. Use of systemic antibiotics
C. Chest tube insertion
Q-105- the following are the risk factor of perpural infection, Except?
A. Endometriosis
B. Cervical leceration
C. Hemorrhage
D. Anemia
E. Retained placenta
Q-106- epdidymitis,one is true.?
A. The peak age b/w 12-18 yrs.
B. U/S is diagnostic
C. Scrotal content within normal size
D. Typical iliac fossa pain.
E. Non of the above
Q-107-All are the complication of laproscopic cholecystectomy, except?
A. Incisional hernia
B. Persistent pneumo-peritonitis
C. Bile leakage
D. Ascietis
Q-108- Post laproscopic cholecystectomy pt present with progressive jaundice, the most appropriate investugation is?
A. ERCP
B. IV cholengiography
Q-109- All of the following organism cause diarrhea with invasion, Except?
A. Shigilla
B. Yersinia
C. Salmonella
D. Cholera
E. Campylobacter
Q-110- All are true about congenital squint, Except?
A. -There is no difference of the angle of deviation of squint eye b/w far & near sightness
B.
C. -
D. -
Q-111- good prognostic factor for pt with schizophrenic is?
A. +ive family history..
B. no previous cause
C. gradual onset
D. prominent affective symptom
E. flat mood
Q-112- A child attend the clinic 3 time with H/O cough for 5 days, not respond to symptomatic treatment, one is true in management?
A. CXR is mandatory
B. Trial of bronchodilater
C. Trial of antibiotics
D. -
E. -
Q-113- BPH, all are true, Except
A. prostitis
B. nocturia
C. diminished size & strength of stream
D. hematuria
E. urine retension.
Q-114- Which of the following is not true regarding osteomylitis?
A. Pneumocystitis
B. Epphyseal plate distruction
C. Septic arthritis
D. Septicemia
E. Further bone growth
Q-115-varicose vein will affect all of the following Except?
A. Short sephanous vein
B. Long sephanous vein
C. Popliteal veinperforators
D. -
Q-116- Retinal detachment, all of the following are true, Except?
More common in hyper-metropic pt than myopic
A. -
B. -
C. -
Q-117- all of the following muscles are rotator cuff, Except?
A. Supra-spanatus
B. Teris-minor
C. Infra-spinatous
D. Deltoid
E. -
Sample-3
Q-1- Complication of systemic hypertension, except?
A. -intra-cerebral H/gs
B. -renal artery stenosis
C. -
D. -
E. -
Q-2- S3 occur in all of the following Except?
A. -tricuspid regurgitation
B. -young athlete
C. LV failure
D. Mitral stenosis
E. -
Q-3- Treatment of chronic atrial fibrillation all except?
A. Cardioversion.
B. Digoxin
C. -Warfarin
D. -
E. -
Q-4- Treatment of unstable angina all except?
A. Heparin
B. IV nitroglycerin
C. -beta-blocker
D. -asprin
E. -
Q-5- Pt with right femur fracture developed chest pain, hemoptysis, ABG PO2 increased, first line of management
a- Hebarin
b- Aminophyllin
c- ii
d- ii
Q-6- Pt with pain in sacroiliac joint, with morning stiffness x-ray of sacroiliac joint …. All will be found EXCEPT:
a- RF-ve
b- Subcutaneous nodule
c- Male .> female
d- ii
Q-7- Fecal leuckocyte in all, EXCEPT:
a- Shigellosis
b- clindamycin induce colitis
c- Idiopathic ulcerative cloilitis
d-
Q-8- Child with TB all found Except?
A. H/O exposure to Tb Pt
B. Chest x-ray finding
C. Splenomagaly
D. AFB ingastric levage
E. -


Q-9- In brucellosis all of the following true, Except?
A. Back pain
B. Hepatomegaly
C. Splenomegaly
D. Lymphadenopathy
E. Gastroenteritis
Q-10- All can be used for treatment of acute gout, Except
A. Allopurinol
B. Penicillamine
C. Gold salt
D. Paracetamol
E. Indomethacin
Q-11- In 6 months old pt with sepsis, most likely organism in infant?
A. Listeria
B. B-hemolytic streptococci
C. H Influenzae type-B
D. Staphlococcus epidermidis
E. -
Q-12- Mycoplasma pneumoniae??
A. Co-agulase +ive
B. Lobar consolidation
C. -
D. -
E. -
Q-13- Community acquired pneumonia treatment is?
A. First generation cephalosporin
B. Penicillin-G + second generation cephalosporin
C. Erythromycin
D. -
Q-14- All vaccines given in KSA for normal child, except?
A. TB
B. Pertusis
C. H Influenza type-B
D. Mumps
E. Diptheria
Q-15- UTI in children
A. Diagnosed by isolation of organism > 100,000 colony count-
B. -
C. -
D. -
E. -
Q-16-Hypokalemia occur in all except?
A. Metabolic alkalosis
B. Acute tubular acidosis
C. Chronic diarrhea
D. Hyper aldosterone
E. Frusamide treatment
Q-17- Urine analysis shows all, except?
A. Phosphate
B. Specific gravity
C. Concentrating capacity
D. Protien
E. Sugar
Q-18- ARF all of the following true Except?
A. Phosphatemia
B. Uremia
C. Increased acid phosphatase
D. Increased K
E. Decreased Na
Q-19- Pt with analytic shock, all of the following are true in treatment Except?
A. Epinephrine
B. Hydralazin
C. Adrenaline
D. Aminophylin
E. Oxygen

Q-20- Patien with partial thicknes burn?
A. Its sensitive
B. Its insensitive
C. It will change to slough within 2-3 wks
D. Need split graft
E. Need free flap
Q-21- 82 yrs old pt with acute urinary retension,management?
A. Empty UB with foleys catheter & followup inlcinic.
B. Insert F.C then send home to come back in clinic
C. Admission & investigation, TURP
D. Immediate prostatectomy.
E. -
Q-22- A 6 yrs old female from Jezan with hematuria, all investigation needed Except?
A. Hb-S
B. Cystoscopy
C. Hb-Electrophoresis
D. Urine analysis
E. U/S abdomen to see changes in Glomerulus
Q-23- Pt with Hb-8, MCV-82, Retic- 10%, needed all, Except?
A. Hb-Electrophoresis
B. Coombs test
C. Serum iron
D. Serumbilirubin
E. -
Q-24- A boy with coca cola color urine, 3 wks back a throat swab showed group-A B-hemolytic streptococci, in favour of diagnosis of post strep GN?
A. RBC cast in urine analysis
B. Increased creatinine
C. Streptozyme test
D. Decreased complement
E. Shrunken kidney by U/S
Q-25- Pt with wt loss all true except?
A. thyrotoxicosis
B. Nephrotic synd
C. TB
D. Aids
E. Carcinoma
Q-26- A 36 yrs old female with FBS-14 mmol & glycosuria, without ketoneuria, treatment is?
A. Intermittent I/M insulin NPH
B. Sulphonylurea + diabetic diet
C. Diabetic diet only
D. Metformin
E. -
Q-27- A 50 yrs old female with rectal bleeding, on examination external hemorrhoid, traetment?
A. Advised excision of hemorrhoid
B. Do nothing & followup in 6 months
C. Sent home on iron tabbowel
D. enema + colonoscopy
E. Rigid sigmoidoscopy
Q-28- Peritonitis
A. pt roll over with agony[pain]
B. pt lies still
C. decreased pulse arte
D. -
E. -
Q-29- Cord propla[se occur in all, except?
A. Premature rupture of membranes
B. Preterm delivery with rupture of membrane
C. Oligohydromnious
D. Head high in pelvis
E. -
Q-30- Diabetes in pregnancy?
A. Oligohydromnious
B. Hypoglycemia ccur in baby after delivery
C. Hyper-calcemia in the baby
D. -
E. -
Q-31- Stress ulcer can be found in all, except?
A. Burn
B. Asprin
C. CNS lesion
D. Penecillin
E. -
Q-32- Peripheral neuropathy can occur in all, except?
A. Lead poisoning
B. DM
C. Gentamycin
D. INH [Anti-TB]
E. -
Q-33- Pt with upper abdominal pain, all in favour of peptic ulcer, Except?
A. Hunger pain
B. Heart burn
C. Epigastric mass
D. epigastric tenderness
E. hematemesis
Q-34- premalignant lesion?
A. Pedunculated polyp
B. Villous papilloma
C. Polypoid polyp
D. Juvenile polyp
E. -
Q-35- Multiple ulcer of medial aspect of the leg with redness & tenderness around it, most likely?
A. Venous ulcer
B. Ischemic ulcer
C. Carcinoma
D. -
E. -
Q-36- 35 yrs female with bloody discharge from nipple, on examination cystic swelling near areols?
A. Duct ectasia
B. Intra ductal papilloma
C. Fibro-adenoa
D. -
E. -
Q-37- Appendicitis most diagnostic?
A. Fever
B. Diarrhea
C. Urinary symptom
D. Leukocytosis
E. Tender right lower quadrant with rebound
Q-38- Pt known to have gall stones presented with central abd pain & bruisng in the flank, diagnosis?
A. Acute cholecystitis
B. Acute pancreatitis
C. Acute appendicitis
D. Acute peritonitis
E. -
Q-39- Congenital hip dislocation?
A. Diagnosed after 3 yrs
B. Abduction + flexion [ortolani test] produce click
C. Abduction not limited
D. Lenghtening of the leg
E. Treatment by open reduction

Q-40- supracondylar fracture pt presented with swelling & cynosis of finger after plaster, management
A. removal of plint near finger.
B. Entire removal of all splint.
C. -
D. -
E. -
Q-41- adduction hip & Internal rotation in fixed position will be?
A. Anterior dislocation of hip
B. Posterior dislocation of hip
C. -
D. -
E. -
Q-42- Neonate just dilivered, term pregnancy, developed respiratory distress CXR multiple lesion in left side shifted mediastinum to the right, decreased bilateral breath sound & flat abdomen?
A. Diaphragmatic hernia
B. RDS
C. Emphysema
D. -
E. -
Q-43- A 2 months boy with projectile vomitting, on examination olive mass in right upper quadrant of abdomen first step of investigation?
A. X-ray abdomen
B. Urea & electrolyte
C. Barium studies
D. -
Q-44- 2 days old neonate presented with peri rectal bleeding, diagnosis?
A. Mickles diverticulum
B. Intussception
C. Fissure-in -ano
D. -
E. -
Q-45- Child with imperforated anus the most useful diagnostic procedure?
A. Plane X-ray abdomen with child inverted position
B. Plane X-ray abdomen
C. -
D. -
E. -
Q-46- 18 months old pt, the mother complain that pt is saying only mama & baba, no other word pt otherwise completely normal first step to evaluate?
A. Physical examination
B. Hearing test
C. Developmental test
D. Test speech
E. -
Q-47- Perinatal asphyxia could be caused by all, Except?
A. Abruptio placentae
B. Hyperemesis gravidarum
C. Pre-eclempsia
D. -
E. -
Q-48- S/S of normal pregnancy, Ecept?
A. Hyperemesis
B. Hega’s sign
C. Clawic sign
D. Amenorrhea
E. -
Q-49- In twin all ture, Except?
A. Dizygote more common than monozygote
B. In dizygote more twin-to-twin transfusion
C. Physical change double time than single form
D. U/S can show twins
E. -

Q-50- ectopic pregnancy, Except?
A. Occur ovarian in 20%
B. Empty uterus by U/S with high beta HCG before 12 wks
C. Beta-HCG double of normal
D. -
E. -
Q-51- Breech presentation all true, Except?
A. Breech after 36 wks about 22%
B. Known to cause intra cranial hemorrhage
C. Known with prematurity
D. -
E. -
Q-52-In lactation all true, Except?
A. Sucking stimulate prolactin
B. Sucking cause release of oxytocin
C. Milk releasedecreasedby over hydration
D. -
E. -
Q-53- Secondary amenorrhea?
A. Always pathological
B. Is a part of sheehan syndrome
C. turner syndrome
D. -
E. -
Q-54- Pt with PPH & infertility, all can be found Except?
A. Balloning of sella turcica
B. Decreased Na
C. Hypoglycemia
D. Decreased T4
E. Decreased iodine uptake
Q-55- Placenta previa, all are true Except? Shock out proportion of bleeding
A. Malpresentation
B. Head not engaged
C. Ppainless bleeding
D. -
Q-56- PID pelvic inflamatory disease, all are true, Except?
A. Infertility
B. Endometriosis
C. Dysparunia
D. Can be traeted surgically
E. -
Q-57- Laproscopy could be used in all, Except?
A. Infertility
B. Intestinal obstruction
C. Primary amenorrhea
D. -
E. -
Q-58-Recurrent abortion, except?
A. Genetic abnormality
B. Uterine abnormality
C. Thyroid dysfunction
D. Dmincreased prolactin
Q-59- DIC occur in all,Except?
A. Abruptio placenta
B. Fetal death
C. DM
D. Pre-eclampsia
E. -
Q-60- PIH, all are true, Except?
A. Ankle edema
B. Polyuria
C. Exaggerated reflex
D. Right upper quadrant
E. -

Q-61-pyelonephritis in pregnancy all are true, Except?
A. Gentamycin is a drug of choice
B. Abruptio placenta should be ruled out
F. coli is a commoc organism
C. Should be traeted even for asymptomatic
D. -
Q-62- All indicat severity os bronchial asthama, Except? Intercostal & supraclavicular retraction
A. Exhaustion
B. PO2 60 mmHg
C. PO2 60 mmHg +PCO2 45 mmHg
D. Pulsus paradoxis > 20 mmHg
Q-63- All are normal in association with teething Except?
A. Rinorrhea
B. Diarrhea
C. Fever > 39 C
D. Irritability
E. -
Q-64- Pt come within 3 hrs with H/O lf side weakness, on examination revealed lf side hemiparesis, pulse 120 min, irregular with diastolic murmur at mitral area first step of management?
A. Heparin
B. Digoxin
C. EEG
D. Cardioangiography
E. ECG
Q-65- Pt with fever, pallor petechei, CBC as WBC-2,800/cmm2, Hb-6 gm%, plt- 29,000,
next step of investigation?
A. B/M aspiration
B. -
C. -
D. -
Q-66- Pt on chemotherapy presented with fever, all should be done, Except?
A. Blood C/S
B. Urine C/S
C. Asprin is effective
D. Broad spectrumantibiotic
E. -
Q-67- Chrons disease indicationofsurgery is?
A. Intestinal fistula
B. External fistula
C. Intestinal obstruction
D. Abd mass
E. -
Q-68- In infected index finger,all can be use, except?
A. Rubber tourniquet
B. Xylocaneadrenalin
C. Ring block
D. -
E. -
Q-69- Pt with low grade fever & arthralgia for 5 days, presented with pansystolic murmur at apex, H/O difficulty in swallowing with fever 3 wks back, most likely diagnosis is?
A. Bacterial endocarditis
B. Viral myocarditis
C. Acute rheumatic fever
D. Pericarditis
E. -
Q-70- Signs of CCF in children all, except?
A. Gallop rhythm
B. Periorbital edema
C. Basal crept
D. Hepatomegaly
E. Bounding pulse
Q-71- 7 months old infant with 4 m H/O interruption of feeding, normal S1 loud S2 pansystolic murmur grade III / IV at 3rd lf intercostal space parasternally, with hyperactive pericardium, diagnosis?
A. Large VSD
B. Large ASD
C. PDA
D. Aortic regurgitation
E. Mitral regargitation
Q-72- Pt present with fever swelling is felt,anterior LN, swelling warm, tender & fluctant, diagnosis?
A. Viral infection
B. Bacterial lymphadenitis
C. Hodgkin lymphoma
D. ALL
E. -
Q-73- A 50 yrs old female pt with H/O wt loss, preference of cold weather, palpitation, there is
H/O firm swelling in the anterior neck for 5 yrs, diagnosis is?
A. Simple goiter
B. Diffuse toxic goiter[grave disease]
C. Carotid body tumor
D. Parathyroidedema
Q-74- Pt presented with fluctuant redness of finger bulb, traetment?
A. Incision
B. Penicillin
C. -
D. -
E. -
Q-75- Infertility, all true, except?
A. Male factor present 24%
B. Normal semen analysis is > 20%
C. Idiopathic infertility is 27%
D. High prolactin could be a cause
E. -
Q-76- Pt with H/O fever, peripheral blood film +ive for malaria?
A. Banana shaped erythrocyte is seen in P.vivax
B. Mostly due to P Falciparum
C. Treated immediately with primaquine 10mg for 3 days
D. Response to treatment will take 72hrs to appear.-
E. -
Q-77- Primary amenorrhea due to?
A. Failure of canalization of mullarian duct
B. Kallman syndrome
C. Agenesis
D. All of the above
E. Non of the above
Q-78- Pt with oliguria, one contraindicated?
A. IV ringer lactate
B. IVP
C. -
D. -
E. -
79-Pt with multiple truma, conscious, treatment?
A. ABC
B. IVF
C. Cross match-
D. -
E. -
Q-80- Porat HTN the least cause hepatocellular damage?
A. Schistosomiasis
B. Alcohol cirrgosis
C. Post necrotic scaring
D. Cirrhosis due to chronic active hepatitis
E. -
Q-81- Obstructed labor?
A. Primigravida
B. Easy to diagnosed early before starting labor
C. -
D. -
E. -
Q-82- RTA with urethral bleeding, step of management?
A. Insert foleys catheter.
B. Stabilized the pelvis
C. Inser suprapubic catheter.
D. -
E. -